[obm-l] RE: [obm-l] Problema do Márcio - jogo de tv

2002-10-14 Por tôpico João Gilberto Ponciano Pereira

1) Sabemos que 2^n = soma(2^(n-1) + 2^(n-2)... 2^0) + 1

2) seja n o número da pergunta de maior número de pontos (começando pela
pergunta zero!), e T o total de pontos obtidos com as respostas, de forma
que:
T = 2^n + resto, com resto positivo, na forma soma(2^(n-1) + 2^(n-2)...
2^0). Como sabemos que 2^n  resto  ==  2^(n+1)  2^n + resto, podemos
concluir que podemos definir n como 2^(n+1)  T = 2^n, ou seja, a maior
potência de 2 que cabe dentro de T. Para definirmos a segunda resposta,
pasta fazer T' = T - 2^n e fazer o mesmo raciocínio.

-Original Message-
From: Gabriel Pérgola [mailto:[EMAIL PROTECTED]]
Sent: Sunday, October 13, 2002 8:06 PM
To: Obm-l
Subject: [obm-l] Problema do Márcio - jogo de tv


E aí pessoal,

Estava olhando o problema que o Márcio mandou para a lista:

Em um jogo de televisão, um candidato deve responder a 10 perguntas. A
primeira vale 1 ponto, a segunda vale 2 pontos, e assim, sucessivamente,
dobrando sempre. O candidato responde a todas as perguntas e ganha os pontos
correspondentes às respostas que acertou, mesmo que erre algumas. Se o
candidato obteve 610 pontos, quantas perguntas acertou?


E vi a solução usando número binários (colocando na base dois)..

Gostaria de saber se existe alguma outra forma de resolver este problema, e
se sim, como?

Abraço,

Gabriel

=
Instruções para entrar na lista, sair da lista e usar a lista em
http://www.mat.puc-rio.br/~nicolau/olimp/obm-l.html
O administrador desta lista é [EMAIL PROTECTED]
=
=
Instruções para entrar na lista, sair da lista e usar a lista em
http://www.mat.puc-rio.br/~nicolau/olimp/obm-l.html
O administrador desta lista é [EMAIL PROTECTED]
=



RE: [obm-l] Teorema de Donald

2002-10-18 Por tôpico João Gilberto Ponciano Pereira
Pessoal
 
E se pensarmos mais ou menos assim:
 
(1) O número de diferentes combinações de casais num salão com K homens e K
mulheres será K!.
 
(2) Seja o casal (H1, Ma) (H2, Mb). Sabemos que se a dupla de casal é:
 
a-) Estável: Logo, (H1, Mb) (H2, Ma) é uma dupla Instável
b-) Instável: Logo, (H1, Mb) (H2, Ma) é uma dupla Estável.
 
Ou seja, se formos analisar todas as combinações possíveis de duplas de
casais (H1, Mx) (H2, My), sabemos que metade destas combinações são válidas
(estáveis)!
 
Minha idéia era provar por combinação que existe pelo menos uma das K!
combinações que é válida, fazendo este raciocínio para todos os pares de
casais.
 
De cara, fazendo este raciocínio para H1 e H2, já eliminamos metade das
combinações!
 
O que acham??
 
JG

-Original Message-
From: Domingos Jr. [mailto:dopikas;uol.com.br]
Sent: Thursday, October 17, 2002 7:50 PM
To: [EMAIL PROTECTED]
Subject: Re: [obm-l] Teorema de Donald


Um algoritmo feito assim pode varrer todas as combinações de pares possíveis
se isso for necessário (acho que é altamente improvável que isso ocorra,
talvez seja até impossível).
 
a demonstração que eu tinha imaginado para o problema (logo após a sua
primeira postagem) era +/- assim:
 
para n = 1 (1 casal) temos que é sempre possível obter um 1-casamento
suponha para todo 1 = n  k seja sempre possível obter um n-casamento
um (n+1)-casamento pode ser obtido da seguinte maneira:
1) Existe um subconjunto Hx (1= x = n), de x homens, e Mx, de x mulheres,
tal que para todo homem de Hx as x mulheres preferidas estão todas em Mx e
para toda mulher de Mx seux x homens preferidos estão em Hx.
neste caso, podemos dividir o problema em dois menores, arranjando um
x-casamento e um (n + 1 - x)-casamento, o que é perfeitamente possível
dentro da hipótese de indução
 
2) há uma mistura total de gostos!, neste caso estamos com um problema chato
para resolver.
 
ainda não tive nenhuma boa idéia de como lidar com esse caso.
 

- Original Message - 
From: Johann Peter Gustav Lejeune  mailto:peterdirichlet2002;yahoo.com.br
Dirichlet 
To: [EMAIL PROTECTED] mailto:obm-l;mat.puc-rio.br  ;
[EMAIL PROTECTED] mailto:teoremalista;yahoogrupos.com.br  ;
[EMAIL PROTECTED] mailto:teoremaprob;yahoogrupos.com.br  
Sent: Thursday, October 17, 2002 3:59 PM
Subject: [obm-l] Teorema de Donald


Para quem nao leu minha mensagem nem se tocou de que estou vivo :):)

A demonstraçao sera assim:Escolha um homem M1,e noive com a predileta M1.

ALGORITMO:

1)Escolha um homem solteiro.

2)Verifique quem e sua predileta,e

2') faça com que ele a proponha em noivado.

3)Se a moça for solteira,ela aceita incondicionalmente;caso ela esteja
noivada,ela escolhe,dentre seu noivo e o proponente,o mais bem colocado da
sua lista de preferencias.

4)Deste passo,pode ocorrer:

4.1-A moça pretendida e solteira.Deve-se entao retornar a 1).

4.2a-A moça pretendida e noivada e o rejeita.Neste caso ele procura pela sua
segunda predileta,e volta a 2')

4.2b-A moça pretendida e noivada e o aceita.Assim sendo seu ex-noivo age
como em 4.2a.

Quando o processo acabar trealiza-se um n-casamento.

Agora dei o exercicio de bandeja:PROVE QUE ISTO DA CERTO.

 

Te mais galera!!!



TRANSIRE SVVM PECTVS MVNDOQUE POTIRE

CONGREGATI EX TOTO ORBE MATHEMATICI OB SCRIPTA INSIGNIA TRIBVERE

Fields Medal(John Charles Fields)




  _  

Yahoo! GeoCities http://br.geocities.yahoo.com/ 
Tudo para criar o seu site: ferramentas fáceis de usar, espaço de sobra e
acessórios.

=
Instruções para entrar na lista, sair da lista e usar a lista em
http://www.mat.puc-rio.br/~nicolau/olimp/obm-l.html
O administrador desta lista é [EMAIL PROTECTED]
=



RE: [obm-l] desafio !

2002-11-04 Por tôpico João Gilberto Ponciano Pereira
Um comandante de companhia convocou voluntários para a constituição de 11
patrulhas. Todas elas são formadas pelo mesmo número de homens. Cada homem
participa de exatamente duas patrulhas. Cada duas patrulhas tem somente um
homem em comum. Determine o múmero de voluntários e integrantes de uma
patrulha. 
 
Pessoal
 
E se tivermos apenas 11 homens, com 2 homens por patrulha, de forma que
P(1) = H(1) e H(2)
...
P(N) = H(N) e H(N+1)
P(11) = H(11) e H(1)
 
SDS
JG

-Original Message-
From: Eder [mailto:edalbuquerque;uol.com.br]
Sent: Sunday, November 03, 2002 1:03 PM
To: [EMAIL PROTECTED]
Subject: Re: [obm-l] desafio !


Uma vez alguém me falou de uma analogia interessante que poderia ser
utilizada neste problema...
É o seguinte:
 
Consideremos um polígono convexo de 11 lados e,é claro,de 11 vértices.Você
posicionaria 1 homem em cada vértice,assim estes estariam em exatamente duas
patrulhas e cada duas patrulhas teriam um homem em comum.Agora, considere as
diagonais.Imaginando uma diagonal como um homem,vemos que aqui também é
respeitado o fato de que cada homem estaria em exatamente duas patrulhas e
cada duas patrulhas,ligadas pela diagonal,teriam um homem em comum.Assim o
total de homens é:
 
nº vértices+nº diagonais= 11 +11(11-3)/2 = 55.
 
 
Seja n o número de homens por patrulha.Temos que 11*n=2*55 (pois cada homem
foi contado duas vezes),daí n=10.
 
Espero que esteja certo.
 
 
Eder
 

- Original Message - 
From: Wander  mailto:wander29br;hotmail.com Junior 
To: [EMAIL PROTECTED] mailto:obm-l;mat.puc-rio.br  
Sent: Sunday, November 03, 2002 2:09 PM
Subject: [obm-l] desafio !

Dúvida:
 
Um comandante de companhia convocou voluntários para a constituição de 11
patrulhas. Todas elas são formadas pelo mesmo número de homens. Cada homem
participa de exatamente duas patrulhas. Cada duas patrulhas tem somente um
homem em comum. Determine o múmero de voluntários e integrantes de uma
patrulha.
 
 
Agradeço desde já.
Wander

=
Instruções para entrar na lista, sair da lista e usar a lista em
http://www.mat.puc-rio.br/~nicolau/olimp/obm-l.html
O administrador desta lista é [EMAIL PROTECTED]
=



[obm-l] RE: [obm-l] Re: [obm-l] Re: [obm-l] G. Analítica - Área de Polígonos

2002-12-19 Por tôpico João Gilberto Ponciano Pereira
Pessoal

Sem querer ser chato, mas acho que jeito muito mais simples de demostrar
isso.

1- Vamos imaginar um polígono de N vértices, ordenados no sentido horário.
Considere o vértice N+1 = vértice 1

2- Agora, vamos chamar de Tn o trapézio formado pelos vértices (Xn,
Yn)(Xn+1,Yn+1)(Xn+1,0)(Xn,0). A área pode ser calculada por (Xn+1 - Xn) *
(Yn+1 + Yn) / 2. (Vale área negativa)

3- Graficamente, dá para ver que SOMA(área(Tn)) n de 1 a N é a área do
polígono em questão.

Desenvolvendo a soma, temos Área = (Soma(Xn * Yn+1) - Soma(Xn+1 * Yn)) / 2.

Detalhe: Vale para qualquer polígono, desde que suas arestas não se cruzem.

-Original Message-
From: Marcio [mailto:[EMAIL PROTECTED]]
Sent: Wednesday, December 18, 2002 6:15 PM
To: [EMAIL PROTECTED]
Subject: [obm-l] Re: [obm-l] Re: [obm-l] G. Analítica - Área de
Polígonos


Bom, ja falei isso numa mensagem anterior, mas vou assumir que as
pessoas nao leram pq o titulo da msg acabou ficando estranho...
Eh obvio que esses macetes podem ser demonstrados.. Acho que ainda
teremos que estudar  muita matematica ateh q a gente comece a precisar
utilizar resultados fortes ainda em aberto. A demonstracao por inducao eh
bem natural. Vou colocar aqui.
(i) Caso n =3 (eh o + dificil):
Se os vertices sao (xi,yi), i=1,2,3 (tomados no sentido anti-horario),
entao a área é:
2S = absenx = |(x2-x1, y2-y1) X (x3-x1, y3-y1)| = ... (Se vc nao sabe que o
modulo do produto vetorial eh absenx, vc pode recriar isso, elevando os dois
lados da 1a eq. ao quadrado e usando a lei dos cossenos para ficar com uma
expressao que soh dependa dos lados.. eh grande mais eh rapido. Pensando
mais um pouco, ve-se que se os vertices estao no sentido anti horario o
determinante sempre da positivo).

(ii) Suponha que qualquer poligono convexo com n ou menos vertices, tem
area dada por (x_1*y_2 + x_2*y_3 + ... + x_n-1 *y_n + x_n*y_1 - y_1*x_2 -
... - y_n*x_1)/2 (onde os vertices (xi,yi) foram tomados no sentido
anti-horario).
Dado um poligono convexo de n+1 vertices A0, A1, ... An, vc pode
calcular sua area somando as areas do poligono A1-...-An com a do triangulo
A0A1An (note que aqui eu estou usando a convexidade). Usando a hip. de
inducao, o dobro da area desse poligono eh:
(x1*y2 + x2*y3 + ... + x_n-1 *yn + 'xn*y1' - y1*x2 - ... - 'yn*x1')+
(x0*y1+'x1*yn' + xn*y0 - y0*x1 - 'y1*xn' - yn*x0) =
(x0*y1 + x1*y2 + ...+ xn*y0  -  y0*x1 - y1*x2 - ... - yn*x0), ou seja o
resultado tmb vale para n+1 vertices..

O passo mais dificil dessa demonstracao eh vc inicialmente modificar um
pouco o resultado que as pessoas costumam conhecer. Ao inves de supor que a
area eh dada pelo modulo do determinante (no caso geral), vc supoe que ela
eh o proprio det, desde que os vertices sejam tomados no sentido
anti-horario.

Um resultado analogo a esse pode ser estabelecido na notacao dos numeros
complexos. Existe um problema interessante, resolvido na Eureka

Marcio

PS: Na minha (modesta) opiniao, eh muito mais saudavel vc usar esse metodo
numa prova do que usa-lo no rascunho e simplesmente escrever a resposta
dizendo que dividiu o poligono em triangulos.

- Original Message -
From: Bruno Lima [EMAIL PROTECTED]
To: [EMAIL PROTECTED]
Sent: Wednesday, December 18, 2002 5:13 PM
Subject: Re: [obm-l] Re: [obm-l] G. Analítica - Área de Polígonos


 Eu tambem conheco o tal macete, mas nunca consegui
 justifica-lo. Me parce que ele só serve para poligonos
 convexos.


  --- Igor GomeZZ [EMAIL PROTECTED] escreveu: 
  Em 18/12/2002, 09:09, Wagner ([EMAIL PROTECTED])
  disse:
 
   Ola para todos !
 
   De jeito nenhum! Nunca se deve colocar em uma
  dissertativa um
   determinante de uma matriz não quadrada.
   Você pode usar esse método no rascunho e dizer que
  dividiu o polígono em
   triângulos e somou a área deles.
   Outra coisa, explique melhor o macete pois ele não
  é tão conhecido.
 
  Blz Wagner, acabei de mandar a explicação...
 
  Vc faz parte de alguma banca de correção? (eh o
  Wagner dos livros da SBM?)
 
  Eu queria saber se existe alguma prova no curso
  superior, algo dentro de
  Algelin por exemplo...
 

=
Instruções para entrar na lista, sair da lista e usar a lista em
http://www.mat.puc-rio.br/~nicolau/olimp/obm-l.html
O administrador desta lista é [EMAIL PROTECTED]
=
=
Instruções para entrar na lista, sair da lista e usar a lista em
http://www.mat.puc-rio.br/~nicolau/olimp/obm-l.html
O administrador desta lista é [EMAIL PROTECTED]
=



RE: [obm-l] analise combinatoria

2002-12-19 Por tôpico João Gilberto Ponciano Pereira
Resposta: 2 cores

Pegue um ponto de uma região qualquer e veja quantos círculos contém este
ponto. Se for um número ímpar, pinte de verde. Se for par, de amarelo.

Prova: Basta analisar duas regiões de fronteira. São delimitadas por uma
circunferência, ou seja, uma região possui N e a outra N+1 circunferência
que as contém!

-Original Message-
From: Rafael [mailto:[EMAIL PROTECTED]]
Sent: Thursday, December 19, 2002 2:53 PM
To: OBM
Subject: [obm-l] analise combinatoria


Segue uma questão que caiu na segunda fase do
vestibular da Universidade Federal de Pernambuco em
1999. Se alguém souber o porque da resposta...

07. A figura abaixo contém seis círculos. Um designer
pretende colorir as regiões em que fica dividido o
círculo maior de forma que regiões tendo um mesmo arco
de circunferência como fronteira sejam coloridas com
cores diferentes. Assinale o número mínimo de cores a
serem utilizadas.

(segue a figura circulos.gif anexada)

Grato,

Rafael.

___
Busca Yahoo!
O melhor lugar para encontrar tudo o que você procura na Internet
http://br.busca.yahoo.com/
=
Instruções para entrar na lista, sair da lista e usar a lista em
http://www.mat.puc-rio.br/~nicolau/olimp/obm-l.html
O administrador desta lista é [EMAIL PROTECTED]
=



RE: [obm-l] IME-95

2002-12-27 Por tôpico João Gilberto Ponciano Pereira

   6 esferas identicas de raio R encontram-se posicionadas
 no espaço de tal forma que cada uma delas seja tangente a exatamente 4
 esferas.Desta forma,determine a aresta do cubo que tangencie todas as
 esferas.

Não é óbvio se esta é a única configuração que satisfaz o enunciado;
aliás eu nem tenho certeza se é ou não. Tenho a impressão de que nenhuma
das soluções publicadas considerou esta questão.

Só complementando a resposta do Nicolau, a configuração das esferas é única.
Pelo enunciado, podemos concluir que se E1(esfera 1) e E2 não são tangentes,
E3, E4, E5, E6 são tangentes a ambas. Como todas tem o mesmo raio,
conclui-se que C3(centro de E3)...C6 pertencem um plano de simetria. Podemos
fazer isso para os outros 2 pares restantes. Daí não é difícil provar que os
3 planos são ortogonais entre si, e que a configuração é única. 

Abraços!




-
Outras pessoas já mandaram soluções mas acho que elas eram incompletas
de modo que quero comentar mesmo assim.

Uma possível posição para as 6 esferas é que seus centros ocupem
os vértices de um octaedro regular: (+-c,0,0), (0,+-c,0), (0,0,+-c).
Verificamos facilmente que a distância entre dois centros é
2R =  c sqrt(2) donde c = sqrt(2) R.

Não é óbvio se esta é a única configuração que satisfaz o enunciado;
aliás eu nem tenho certeza se é ou não. Tenho a impressão de que nenhuma
das soluções publicadas considerou esta questão.

Aceitando que as seis esferas ocupem as posições descritas, existem
pelo menos oito cubos que tangenciam as seis esferas: os oito cubos
têm as arestas paralelas aos eixos e vértices e escreverei
([x1,x2],[y1,y2],[z1,z2]) para denotar o cubo de vértices
(x1,y1,z1), (x1,y1,z2), (x1,y2,z1), (x1,y2,z2),
(x2,y1,z1), (x2,y1,z2), (x2,y2,z1), (x2,y2,z2).

Os oito cubos são
([-c-R,c+R],[-c-R,c+R],[-c-R,c+R]) com aresta (2+2 sqrt(2))R
(este parece ser o cubo encontrado nas outras soluções)
([-c-R,c-R],[-c-R,c-R],[-c-R,c-R])
([-c-R,c-R],[-c-R,c-R],[-c+R,c+R])
([-c-R,c-R],[-c+R,c+R],[-c-R,c-R])
([-c-R,c-R],[-c+R,c+R],[-c+R,c+R])
([-c+R,c+R],[-c-R,c-R],[-c-R,c-R])
([-c+R,c+R],[-c-R,c-R],[-c+R,c+R])
([-c+R,c+R],[-c+R,c+R],[-c-R,c-R])
([-c+R,c+R],[-c+R,c+R],[-c+R,c+R]) todos com aresta 2 sqrt(2) R
([-c+R,c-R],[-c+R,c-R],[-c+R,c-R]) com aresta (- 2 + 2 sqrt(2)) R

Note que não foi dito se as tangências eram internas ou externas.
Também não é óbvio se existe algum outro cubo satisfazendo o enunciado
(possivelmente com as arestas não paralelas aos eixos) mas eu suspeito
que não.

Em todo caso há pelo menos três respostas coerentes com o enunciado:
(2+2 sqrt(2))R, 2 sqrt(2) R, (-2+2 sqrt(2))R.

[]s, N.
 
=
Instruções para entrar na lista, sair da lista e usar a lista em
http://www.mat.puc-rio.br/~nicolau/olimp/obm-l.html
O administrador desta lista é [EMAIL PROTECTED]
=
=
Instruções para entrar na lista, sair da lista e usar a lista em
http://www.mat.puc-rio.br/~nicolau/olimp/obm-l.html
O administrador desta lista é [EMAIL PROTECTED]
=



RE: [obm-l] Como guardar um octaedro dento de um cubo (era IME-95)

2002-12-30 Por tôpico João Gilberto Ponciano Pereira
Uma pergunta relacionada, interessante e a meu ver mais clara seria
qual o menor cubo que contem todas as seis esferas.
Ou, para simplificar (mas sem alterar a essência da questão)
podemos perguntar qual o menor caixa cúbica dentro da qual podemos
guardar um octaedro regular de aresta dada. Acho que é mais difícil
do que parece.

1- Intuitivamente, estou admitindo que o centro do cubo envolvente é a
origem do plano cartesiano do problema. (Esferas de raio R e centros
(R*sqrt(2),0,0) ). Não sei exatamente como provar, mas se isto não for
verdade, se uma das faces tangencia uma esfera, o lado oposto do cubo não
tangencia a esfera oposta do sistema (a única das outras 5 esferas que não
tangencia a primeira).

2- Estou com a impressão que as soluções para o problema original não são
discretas, mas sim que existe um intervalo contínuo entre o menor e o maior
cubo envolvente. Se imaginarmos r a aresta mínima, conseguiríamos girar o
cubo de aresta r+d de forma que todas as circunferências tocassem as faces
do cubo.

3- Se 2 for verdade, o menor raio seria aquele que impediria a rotação nos 3
eixos. Para que isto ocorra, é necessário que as 3 faces de um dos vértices
do cubo tangenciassem a mesma circunferência. Acho que a aresta para esta
configuração já foi calculada em algum dos e-mails.

-Original Message-
From: Nicolau C. Saldanha [mailto:[EMAIL PROTECTED]]
Sent: Monday, December 30, 2002 12:26 PM
To: [EMAIL PROTECTED]
Subject: [obm-l] Como guardar um octaedro dento de um cubo (era IME-95)


On Fri, Dec 27, 2002 at 06:12:27PM -0300, João Gilberto Ponciano Pereira
wrote:

   6 esferas identicas de raio R encontram-se posicionadas
 no espaço de tal forma que cada uma delas seja tangente a exatamente 4
 esferas.Desta forma,determine a aresta do cubo que tangencie todas as
 esferas.

Esta questão é bem interessante mas também um pouco mal formulada.
Acho que já ficou claro que os centros das esferas são os vértices
de um octaedro regular de aresta 2R. Já vimos que existe mais de um cubo
que tangencia todas as esferas.

Uma pergunta relacionada, interessante e a meu ver mais clara seria
qual o menor cubo que contem todas as seis esferas.
Ou, para simplificar (mas sem alterar a essência da questão)
podemos perguntar qual o menor caixa cúbica dentro da qual podemos
guardar um octaedro regular de aresta dada. Acho que é mais difícil
do que parece.

[]s, N.
=
Instruções para entrar na lista, sair da lista e usar a lista em
http://www.mat.puc-rio.br/~nicolau/olimp/obm-l.html
O administrador desta lista é [EMAIL PROTECTED]
=
=
Instruções para entrar na lista, sair da lista e usar a lista em
http://www.mat.puc-rio.br/~nicolau/olimp/obm-l.html
O administrador desta lista é [EMAIL PROTECTED]
=



[obm-l] OffTopic RE: [obm-l] trigonometria (transformação de arcos)

2003-02-05 Por tôpico João Gilberto Ponciano Pereira
Sugiro alterar o nome da lista de: OBM - Olimpíada Brasileira de Matemática
para: LGE - Lista do Gabarito Errado.

-Original Message-
From: A. C. Morgado [mailto:[EMAIL PROTECTED]]
Sent: Wednesday, February 05, 2003 4:41 PM
To: [EMAIL PROTECTED]
Subject: Re: [obm-l] trigonometria (transformação de arcos)


Voce esta certo. Seu gabarito, como sempre, errado.

[EMAIL PROTECTED] mailto:[EMAIL PROTECTED]  wrote:


(UECE) Se  P= [(sen 40º)/(sen 20º)] - [(cos 40º)/(cos 20º)], então p^2 - 1 é
igual: 
Resp: cotg^2 (20º) 

Obs: Será que o resultado não é tg^2 (20º)? 

ICQ: 337140512


=
Instruções para entrar na lista, sair da lista e usar a lista em
http://www.mat.puc-rio.br/~nicolau/olimp/obm-l.html
O administrador desta lista é [EMAIL PROTECTED]
=



RE: [obm-l] restorno: estudo de sinais

2003-02-05 Por tôpico João Gilberto Ponciano Pereira
(2x^2 + x + 5)/ x=0
 
O numerador vai ser positivo para qualquer x. Logo, o denominador vai
determinar a desigualdade...
x positivo, a desigualdade é falsa.
x=0, indeterminado
x negativo, a desigualdade é verdadeira.
 

 
 
-Original Message-
From: [EMAIL PROTECTED] [mailto:[EMAIL PROTECTED]]
Sent: Wednesday, February 05, 2003 4:57 PM
To: [EMAIL PROTECTED]
Subject: [obm-l] restorno: estudo de sinais


Olá pessoal, 

Eu enviei esta questão: 

(FUVEST) Resolva 2x - 3 + 5*[(1/x) + 1] =1 

resp:{x e R| x0} 

Obs: Eu tentei resolver mas não cheguei neste resultado, vejam minha
resolução e me digam onde errei: 
2x-3+(5/x)+5=1 
2x-3+(5/x)+5-1=0 
2x^2 -3x + 5 + 4x =0 (Nesta etapa eu multipliquei por x) 
2x^2 + x + 5=0 
A partir disso percebe-se que delta é igual -39, portanto não há raízes
reais e a resposta não pode ser :{x e R| x0}. 

Obs: Vocês me disseram que o erro foi que ao invés de 2x^2 + x + 5=0  o
certo seria  (2x^2 + x + 5)/ x=0, portanto temos que x# 0, mas o
discriminante é negativo e sendo assim todo x pertencente a R terá f(x)=2x^2
+ x + 5= e nunca negativo como na resolução. Me dêem uma luz nesta análise
de sinais! 


=
Instruções para entrar na lista, sair da lista e usar a lista em
http://www.mat.puc-rio.br/~nicolau/olimp/obm-l.html
O administrador desta lista é [EMAIL PROTECTED]
=



RE: [obm-l] Tres belos problemas

2003-02-11 Por tôpico João Gilberto Ponciano Pereira
2) Em uma reuniao existem exatamente 201 pessoas de 
5 nacionalidades diferentes. Sabe-se que em cada grupo de 6 pessoas, ao 
menos duas tem a mesma idade. Demonstrar que existem ao menos 5 pessoas do 
mesmo pais, da mesma idade e do mesmo sexo.

Primeiramente podemos distribuir todas as pessoas em apenas 5 grupos de
idade, pois se tivermos 6 grupos, não vale a afirmação Sabe-se que em cada
grupo de 6 pessoas, ao menos duas tem a mesma idade.

Basta utilizar sucessivamente o teorema da casa dos pombos... Ou seja, das
201, sabemos que existe um grupo de 51 pessoas com a mesma idade. Dessas,
sabemos que existe um grupo de 11 pessoas do mesmo país. Dessas, 6 tem o
mesmo sexo.

3) Achei o mais interessante... Vamos dividir o retângulo em 12 quadrados de
lado 1 (4x3). Agora pintamos os quadrados de preto e branco, como um
tabuleiro de xadrez. Se tivermos dois pontos na mesma casa, o problema
está resolvido, pois a distância máxima seria sqrt(2). Se tivermos pontos em
casas vizinha, o problema também está resolvido, pois a distância máxima
seria sqrt(5). Teria que enrolar mais, mas o fato é que os pontos caem ou
todos em casas brancas ou todos em casas pretas. O fato é que existe um
quadrado 3x3 que contém 5 pontos, e novamente pela casa dos pombos, pelo
menos 1 quadrado 1.5 x 1.5 contém 2 ou mais pontos, cuja distância neste
caso é inferior a sqrt(4.5)

-Original Message-
From: Paulo Santa Rita [mailto:[EMAIL PROTECTED]]
Sent: Tuesday, February 11, 2003 1:59 PM
To: [EMAIL PROTECTED]
Subject: [obm-l] Tres belos problemas


Ola Pessoal,

Seguem abaixo tres problemas :

1) Um quadrado e um triangulo estao circunscritos a um circulo de lado 
unitario. Prove que, qualquer que seja a posicao do quadrado e do triangulo,

a area comum aos dois e maior que 17/5. E possivel afirmar que ela e maior 
que 7/2  ?

2) ( Olimpiada Espanhola ) Em uma reuniao existem exatamente 201 pessoas de 
5 nacionalidades diferentes. Sabe-se que em cada grupo de 6 pessoas, ao 
menos duas tem a mesma idade. Demonstrar que existem ao menos 5 pessoas do 
mesmo pais, da mesma idade e do mesmo sexo.

3) ( Olimpiada Russa ) Na regiao delimitada por um retangulo de largura 4 e 
altura 3 sao marcados 6 pontos. Prove que existe ao menos um par destes 
pontos cuja distancia entre eles nao e maior que Raiz_Quad(5).

Estes problemas nao precisam de sugestao.

Um Grande Abraco a Todos !
Paulo Santa Rita
3,1455,110203




_
MSN Messenger: converse com os seus amigos online.  
http://messenger.msn.com.br

=
Instruções para entrar na lista, sair da lista e usar a lista em
http://www.mat.puc-rio.br/~nicolau/olimp/obm-l.html
O administrador desta lista é [EMAIL PROTECTED]
=
=
Instruções para entrar na lista, sair da lista e usar a lista em
http://www.mat.puc-rio.br/~nicolau/olimp/obm-l.html
O administrador desta lista é [EMAIL PROTECTED]
=



[obm-l] RE: [obm-l] RE: [obm-l] RE: [obm-l] Partição

2003-02-20 Por tôpico João Gilberto Ponciano Pereira
Complementando a resposta...

-n, -n+1, -n+2... n-2, n-1, n
-n, -n+1, -n+2... n-2, n-1, n
-n, -n+1, -n+2... n-2, n-1, n

Podemos formar os n+i primeiros trios da seguinte forma:
(-n+i,i,n-2*i), com i de 0 a n. Repare que a soma é zero.

Os últimos n termos são:
(i+n, -n+i -1, n-2*i+1), com i de 1 a n. Mais uma vez, a soma é zero.

Se considerarmos o Item 1, até 2001, temos:

   123 ...  332  333  334 ...  665  666  667
 668  669  670 ...  999 1000 1001 ... 1332 1333 1334
1335 1336 1337 ... 1666 1667 1668 ... 1999 2000 2001

Os trios seriam:
(  1,1000,2001)
(  2,1001,1999)
(  3,1002,1997)
...
(333,1334,1335)

e
(334, 668,2000)
(335, 667,1998)
...
(667, 999,1336)

SDS
JG
-Original Message-
From: João Gilberto Ponciano Pereira [mailto:[EMAIL PROTECTED]]
Sent: Thursday, February 20, 2003 3:43 PM
To: '[EMAIL PROTECTED]'
Subject: [obm-l] RE: [obm-l] RE: [obm-l] Partição


Ops!! Sorry! Parece que entendi o problema de forma errada... Estava fazendo
3 conjuntos de M elementos, e não M conjuntos de 3 elementos, como pedia o
problema Neste caso, temos o seguinte:

A soma total dos elementos do conjunto é 3M *(3M+1)/2. Como os conjuntos de
3 elementos deve ter soma igual, esta soma deverá ser de 3M*(3M+1)/2/M =
Soma = (9M + 1)/2. Já é possível concluir que M é obrigatoriamente ímpar.

Agora, podemos considerar a seqüência com a seguinte nomenclatura:

a(1,-n), a(1,-n+1), ... a(1,0), ... a(1,n)
a(2,-n), a(2,-n+1), ... a(2,0), ... a(2,n)
a(3,-n), a(3,-n+1), ... a(3,0), ... a(3,n)
Com n de -(m-1)/2 a (m-1)/2

Onde a(i,j) = j+(M-1)/2 + M*(i-1) + 1

Se considerarmos que, em cada subconjunto válido, teremos um elemento de
cada linha, temos

a(i1,1) + a(i2,2) + a(i3,3) = (9M+1)/2 = j1 + j2 + j3 + 3*(M-1)/2 + M*((1-1)
+ (2-1) + (3-1)) + 3

Resumindo, temos que j1 + j2 + j3 = 0. Exemplo:

 1  2  3  4  5
 6  7  8  9 10
11 12 13 14 15

fica:
-2 -1 0 1 2
-2 -1 0 1 2
-2 -1 0 1 2

Os conjuntos podem ser:(-2, 1, 1) (-1,-1, 2) ( 0, 2,-2) ( 1, 0,-1) ( 2,-2,
0)
Traduzindo:( 1, 9,14) ( 2, 7,15) ( 3,10,11) ( 4, 8,12) ( 5,
6,13)

Ou seja, temos que provar que, sejam S1, S2, S3 conjuntos idênticos =
(-n...,0,1,2,...n)
Devemos formar 2n+1 trios (um elemento de cada conjunto), tais que a soma do
trio seja zero.

Acho que vale para qualquer n, mas preciso pensar mais um pouco...

-Original Message-
From: João Gilberto Ponciano Pereira [mailto:[EMAIL PROTECTED]]
Sent: Thursday, February 20, 2003 1:23 PM
To: '[EMAIL PROTECTED]'
Subject: [obm-l] RE: [obm-l] Partição


2) Determine todos os inteiros positivos M tais que o conjunto {1 2, ...,
3M} admite uma partição em M subconjuntos de 3 elementos tal que a soma dos
elementos de cada subconjunto é constante.
 
A Questão é... Como distribuir os elementos?
Vamos imaginar uma seqüência de 6 consecutivos
 
n-2, n-1, n, n+1, n+2, n+3. Neste caso, podemos fazer 3 pares de forma que a
soma seja 2n-1:
(n-2) e (n+3)
(n-1) e (n+2)
(n) e (n+1).
 
Logo, se M for par, basta ir distribuindo os números de 6 em 6 (1 a 6, 7 a
12... 3M-6 a 3M), pelo método acima.
 
E se M for ímpar? Neste caso, podemos dividir os 9 primeiros termos (1 a 9)
em 3 grupos de soma igual:
1,5,9 = 15
2,6,7 = 15
3,4,8 = 15
Para o restante, podemos seguir de 6 em 6. (1 a 9, 10 a 15, 16 a 21...1996 a
2001)
 
Proposta: Podemos pensar até num exercício um pouco mais elaborado, do tipo:
 
3) Determine todos os inteiros positivos M tais que o conjunto {1 2, ...,
kM} admite uma partição em M subconjuntos de k elementos tal que a soma dos
elementos de cada subconjunto é constante.

-Original Message-
From: Cláudio (Prática) [mailto:[EMAIL PROTECTED]]
Sent: Thursday, February 20, 2003 12:34 PM
To: [EMAIL PROTECTED]
Subject: [obm-l] Partição


Caros colegas da lista:
 
Estou embananado com este aqui:
 
1) Prove que existe uma partição de {1, 2, ..., 2001} em 667 subconjuntos de
3 elementos tal que a soma dos elementos de cada subconjunto é igual a 3003.
 
2) Determine todos os inteiros positivos M tais que o conjunto {1 2, ...,
3M} admite uma partição em M subconjuntos de 3 elementos tal que a soma dos
elementos de cada subconjunto é constante.
 
Um abraço,
Claudio.

=
Instruções para entrar na lista, sair da lista e usar a lista em
http://www.mat.puc-rio.br/~nicolau/olimp/obm-l.html
O administrador desta lista é [EMAIL PROTECTED]
=
=
Instruções para entrar na lista, sair da lista e usar a lista em
http://www.mat.puc-rio.br/~nicolau/olimp/obm-l.html
O administrador desta lista é [EMAIL PROTECTED]
=
=
Instruções para entrar na lista, sair da lista e usar a lista em
http://www.mat.puc-rio.br/~nicolau/olimp/obm-l.html
O administrador desta lista

[obm-l] RE: [obm-l] Re: RE:_[obm-l]_Partição

2003-02-21 Por tôpico João Gilberto Ponciano Pereira
Ok... Eu errei nas contas... Eu admito! Podem jogar as pedras! (rs)

Claudio, agora ficou fácil a proposta de generalizar o problema. Vou te
deixar pensando algum tempo e depois mando a resposta.

3) Determine todos os inteiros positivos M tais que o conjunto {1 2,
...,kM} admite uma partição em M subconjuntos de k elementos tal que a soma
dos elementos de cada subconjunto é constante.

-Original Message-
From: Cláudio (Prática) [mailto:[EMAIL PROTECTED]]
Sent: Friday, February 21, 2003 2:24 PM
To: [EMAIL PROTECTED]
Subject: [obm-l] Re: RE:_[obm-l]_Partição


Oi, Rafael (e JG):

Bem observado, mas o engano é facilmente remediável. O que importa é que a
idéia do JG foi muito boa (e o que é melhor - produziu uma solução diferente
da que eu achei), mas ele errou nas contas e fez a quebra do conjunto do
meio (668 até 1334) no ponto errado. Ao invés de 1000, ele deveria ter
começado com 1001, de forma que o primeiro trio deveria ser (1,1001,2001), o
segundo (2,1002,1999), etc.

123 ...  332  333  334 ...  665  666 667

668  669  670 ...  999 1000 1001 ... 1332 1333 1334

1335 1336 1337 ... 1666 1667 1668 ... 1999 2000  2001

Os trios corrigidos passam a ser:
(  1,1001,2001)
(  2,1002,1999)
(  3,1003,1997)
...
(334,1334,1335)
e
(335, 668,2000)
(336,669,1998)
...
(667, 1000,1336)

Um abraço,
Claudio.

- Original Message -
From: Rafael [EMAIL PROTECTED]
To: [EMAIL PROTECTED]
Sent: Friday, February 21, 2003 10:57 AM
Subject: RE:_[obm-l]_Partição


 Mas nas conas do JG a soma dos conjuntos dá 2002!!!

 Veja isso de novo:
   (  1,1000,2001) não seria: (1, 1000, 2001)
   (  2,1001,1999)(2, 1001, 1999)
   (  3,1002,1997)(3, 1002, 1997)
   ...
   (333,1334,1335)   (333, 1332, 1337)
 (334, 1333, 1335)???
   e
   (334, 668,2000)
   (335, 667,1998)
   ...
   (667, 999,1336)

 Alguma coisa está errada não?

 Abraços,

 Rafael.



  --- Cláudio_(Prática)
 [EMAIL PROTECTED] escreveu:  Legal!
 
  É uma solução diferente da minha e você foi mais
  técnico do que eu para
  achá-la.
 
  O que eu fiz foi dividir o conjunto {1,2,...,2001}
  em três subconjuntos:
  A1 = {1,2,...,667}
  A2 = {668,669,...,1334}
  A3 = {1335,1336,...,2001}
 
  E começar a formar as partições (lidas
  verticalmente) com cada
  subconjuntinho de 3 elementos recebendo um elemento
  de A1, um de A2 e um de
  A3:
  A1:  0001  0002  0003  ...  0333  0334  0335  0336
  ...  0666   0667
  A2:  1334  1332  1330  ...  0670  0668  1333  1331
  ...   0671  0669
  ou seja, eu coloquei os elementos de A1 em ordem
  crescente de 1 em 1 e os de
  A2 em ordem decrescente de 2 em 2 (mod 2).
 
  Finalmente, eu coloquei os elementos de A3 de modo
  que cada soma fosse igual
  a 3003 meio torcendo pra dar tudo certo, com base
  nos casos especiais (M=3,
  5 e 7) que eu fiz na mão e deram certo.
 
  No entanto, uma vez concluída a partição, é fácil
  ver que tudo daria certo,
  pois a soma dos dois primeiros elementos colocados
  em cada subconjuntinho
  eram todas distintas:
  1335  1334   1333  ...  1003  1002  1668  1667  ...
  1337  1336
  Além disso: 3003 - 1668 = 1335 == todos os
  complementos estavam em A3.
 
  Um abraço,
  Claudio.
 
 
 
  - Original Message -
  From: João Gilberto Ponciano Pereira
  [EMAIL PROTECTED]
  To: [EMAIL PROTECTED]
  Sent: Thursday, February 20, 2003 5:26 PM
  Subject: [obm-l] RE: [obm-l] RE: [obm-l] RE: [obm-l]
  Partição
 
 
   Complementando a resposta...
  
   -n, -n+1, -n+2... n-2, n-1, n
   -n, -n+1, -n+2... n-2, n-1, n
   -n, -n+1, -n+2... n-2, n-1, n
  
   Podemos formar os n+i primeiros trios da seguinte
  forma:
   (-n+i,i,n-2*i), com i de 0 a n. Repare que a soma
  é zero.
  
   Os últimos n termos são:
   (i+n, -n+i -1, n-2*i+1), com i de 1 a n. Mais uma
  vez, a soma é zero.
  
   Se considerarmos o Item 1, até 2001, temos:
  
  123 ...  332  333  334 ...  665  666
  667
668  669  670 ...  999 1000 1001 ... 1332 1333
  1334
   1335 1336 1337 ... 1666 1667 1668 ... 1999 2000
  2001
  
   Os trios seriam:
   (  1,1000,2001)
   (  2,1001,1999)
   (  3,1002,1997)
   ...
   (333,1334,1335)
  
   e
   (334, 668,2000)
   (335, 667,1998)
   ...
   (667, 999,1336)
  
   SDS
   JG


   1) Prove que existe uma partição de {1, 2, ...,
  2001} em 667 subconjuntos
  de
   3 elementos tal que a soma dos elementos de cada
  subconjunto é igual a
  3003.

 ___
 Busca Yahoo!
 O serviço de busca mais completo da Internet. O que você pensar o Yahoo!
encontra.
 http://br.busca.yahoo.com/
 =
 Instruções para entrar na lista, sair da lista e usar a lista em
 http://www.mat.puc-rio.br/~nicolau/olimp/obm-l.html
 O administrador desta lista é [EMAIL PROTECTED

RE: [obm-l] Treta nos quadrados

2003-02-24 Por tôpico João Gilberto Ponciano Pereira
Esta é legal... E acho que sai se fizermos uma análise geométrica do
problema...
 
Vamos chamar a_n = c * bn, de forma que Soma(B_n) = 1 e a_n+1  a_n.
 
Agora vamos imaginar um quadrado de lado 1 onde temos que encaixar os
retângulos (b1,b2), (b2,b3)...
 
Acho que já vi uma solução semelhante, mas com quadrados apenas..

-Original Message-
From: Johann Peter Gustav Lejeune Dirichlet
[mailto:[EMAIL PROTECTED]
Sent: Monday, February 24, 2003 1:10 PM
To: [EMAIL PROTECTED]
Subject: [obm-l] Treta nos quadrados



Essa questao era pra provar algo como 

(a_1+a_2+a_3+...+a_n)^2=4(a_1a_2+a_2a_3+a_3a_4+...+a_n*a_1)

Para o caso n=4 da certo:ab+bc+cd+da=(a+c)(b+d)=((a+c+b+d)/2)^2.

So nao consigo progredir




  _  

Busca Yahoo!  http://br.busca.yahoo.com/ 
O serviço de busca mais completo da Internet. O que você pensar o Yahoo!
encontra.

=
Instruções para entrar na lista, sair da lista e usar a lista em
http://www.mat.puc-rio.br/~nicolau/olimp/obm-l.html
O administrador desta lista é [EMAIL PROTECTED]
=


RE: [obm-l] Problema interessante

2003-02-27 Por tôpico João Gilberto Ponciano Pereira
Tome uma partição QUALQUER de {1,2,...,2n} em dois conjuntos A e B com n
elementos cada.  Ponha os elementos de A em ordem crescente a_1...a_n e os
de B em ordem decrescente b_1...b_n.  Prove que:

|a_1-b_1| + ... + |a_n-b_n| = n^2
 
Acabei esquecendo de mandar a resposta, mas aqui vai ela, espero que em
tempo
 
Vamos chamar os conjuntos:
x = {1 a n} e y={n+1 a 2n}
 
Vamos supor que o conjunto A é formado de tal forma que possua m elementos
de Y e n-m elementos de X. Logo, o conjunto B é obrigatoriamente formado por
m elementos de x e n-m elementos de Y. 
 
Então podemos considerar que:
RESULTADO = |a_1-b_1| + ... + |a_n-b_n| = (a1 - b1) + (a2 - b2) + ... + (am
- bm) - ((am+1 - bm+1) + ... +(an - bn))= 
soma a (1 a m) - soma b(1 a m) - soma a(m+1 a n) + soma b(m+1 a n)
 
Se voltarmos a suposição inicial, soma a (1 a m)  + soma b(m+1 a n) é a soma
dos elementos do conjunto Y, e soma b(1 a m) + soma a(m+1 a n) é a soma dos
elementos de X.
 
Logo, temos que:
RESULTADO = soma(n+1 a 2n) - soma(1 a n) = n*n + soma(1 a n) - soma(1 a n)
=n^2
 
 -Original Message-
From: Cláudio (Prática) [mailto:[EMAIL PROTECTED]
Sent: Monday, February 24, 2003 2:48 PM
To: [EMAIL PROTECTED]
Subject: [obm-l] Problema interessante



Taí um resultado inesperado (pelo menos pra mim):
 
Tome uma partição QUALQUER de {1,2,...,2n} em dois conjuntos A e B com n
elementos cada.  Ponha os elementos de A em ordem crescente a_1...a_n e os
de B em ordem decrescente b_1...b_n.  Prove que:

|a_1-b_1| + ... + |a_n-b_n| = n^2.
 
Um abraço,
Claudio.

=
Instruções para entrar na lista, sair da lista e usar a lista em
http://www.mat.puc-rio.br/~nicolau/olimp/obm-l.html
O administrador desta lista é [EMAIL PROTECTED]
=


[obm-l] RE: [obm-l] Re: [obm-l] Re: [obm-l] Dúvida de vestibular

2003-02-27 Por tôpico João Gilberto Ponciano Pereira
Este é um problema interessante! Mas acho que faltou dizer que as cidades em
questão fazem parte do mesmo país, ou seja, a cidade A pertence a um país C
se existe pelo menos uma estrada que vá de A para alguma cidade pertencente
a C.

Acho que a solução é mais ou menos assim:

A pegadinha é provar que se existe um caminho que vai de Cn para Cm, então
existe o caminho de volta de Cm para Cn. Prova: seja Pi o peso do número
de estradas da cidade Ci, que é o número de estradas que entram mais o
número de estradas que saem. Supondo que o viajante pega o carro de Cm e vai
para Cn passando por algumas cidades. Como ele saiu de Cm, o peso Pm será 1.
Para as outras cidades, o peso é 2i, pois ele chegou e saiu. Para Cn, o peso
é 2i+1, pois ele chegou.

Veja que as cidades devem ter Pi par, pois o número de estradas que chegam é
o mesmo das que saem. Logo, o ciclo de volta sempre terminará em Cm.

Suponha agora um país com 2 cidades A e B. Se A chega a B, temos que B chega
a A, e o número de mapas é igual. Acrescentando uma cidade C no país, ela
será ligada em A, sem perda de generalidade. Ora, se A chega em C e B chega
em A, então B chega em C e, pela volta, C chega em B. Logo, por indução, vc
consegue chegar em qualquer cidade de um determinado país com N cidades, o
número de mapas possíveis é N-1.

-Original Message-
From: Nicolau C. Saldanha [mailto:[EMAIL PROTECTED]
Sent: Thursday, February 27, 2003 1:41 PM
To: [EMAIL PROTECTED]
Subject: [obm-l] Re: [obm-l] Re: [obm-l] Dúvida de vestibular


Eu [ainda] não sei resolver o problema do Okakome mas...

On Wed, Feb 26, 2003 at 04:39:33PM -0300, Domingos Jr. wrote:
  Oi Pessoal,
Estava estudando análise combinatória por uma
  apostila de um curso pré-vestibular, e encontrei o
  seguinte problema, que achei interessante, mas minha
  solução foi muito longa, e não sei se está certa,
  porque tinha muitos casos. Se estivesse num
  vestibular, o que faria?
Num país, as estradas ligam duas cidades e são de
  mão única (pode haver mais de uma estrada entre duas
  cidades). O número de estradas que partem de cada
  cidade é igual ao número de estradas que chegam nessa
  cidade. Um mapa da cidade C é um conjunto de rotas
  que: 1) levam C a cada uma das outras cidades do país,
  sem passar por uma cidade mais de uma vez. 2) Se uma
  rota parte de C a D passando por E, então a rota que
  vai de C a E coincide com o começo da rota de C a D.
  Prove que o número de mapas da cidade C é igual ao
  número de mapas de qualquer outra cidade.
 
 Acho que esse enunciado não está completo:
 - se existe uma cidade com nenhuma estrada partindo ou chegando possui um
 número de rotas 0 e não vai ser igual as demais, até aí é um caso idiota
que
 pode ser excluído do problema.

Neste caso não existe nenhum mapa pois é impossível ligar C a X,
a cidade sem estradas. O problema fica correto pois o número
de mapas é sempre 0.

 - se existem conjuntos de cidades disjuntos ou seja cidades de um
conjunto
 A não possuem rota nenhuma para as cidades do conjunto B e vice-versa:
 
 C1 --- C2 ( 1 estrada de C1 - C2 e outra de C2 - C1  )
 C3 == C4 ( 2 estradas de C3 - C4 e duas de C4 - C3 )
 
 neste caso temos que o número de rotas de N(C1) = N(C2) = 1, mas N(C3) =
 N(C4) = 2.

Também neste caso o número de mapas é sempre 0.

 além disso, há um trecho que eu considero confuso:
 
 2) Se uma rota parte de C a D passando por E, então a rota que vai de C a
E
 coincide com o começo da rota de C a D
 nessa situação parece que a rota de C a E deve ser única, mas podem haver
 outras rotas de C até E sem que uma cidade seja visitada mais de uma
vez...
 mesmo que sempre fossem escolhidos os caminhos que passem por menos
cidades
 isso poderia ocorrer já que é permitido haver mais de uma estrada ligando
 duas cidades.

Acho que ajuda considerar o caso em que todas as estradas são de mão dupla:
neste caso um mapa nada mais é do que uma árvore maximal e o problema
fica trivial.

[]s, N.
=
Instruções para entrar na lista, sair da lista e usar a lista em
http://www.mat.puc-rio.br/~nicolau/olimp/obm-l.html
O administrador desta lista é [EMAIL PROTECTED]
=
=
Instruções para entrar na lista, sair da lista e usar a lista em
http://www.mat.puc-rio.br/~nicolau/olimp/obm-l.html
O administrador desta lista é [EMAIL PROTECTED]
=


RE: [obm-l] Problemas

2003-03-10 Por tôpico João Gilberto Ponciano Pereira
Pessoal

Não sei se foi eu que entendi errado, mas acho que o problema das aranhas é
mais simples:

1) Uma aranha tem uma meia e um sapato para cada uma de suas  8 pernas. De
quantas maneiras diferente a aranha pode colocar as meias e os sapatos,
supondo  que , em cada perna, a meia tem de ser calçada antes do sapato?

As meias e os sapatos são eventos distintos, portanto basta multiplicar  o
número de combinações possíveis de sapatos pelo número de combinações das
meias, ou seja, (n!) ^2. Para o caso humano:

Pé direito  Pé esquerdo
Sapato1 - Meia1 Sapato2 - Meia2
Sapato1 - Meia2 Sapato2 - Meia1
Sapato2 - Meia1 Sapato1 - Meia2
Sapato2 - Meia2 Sapato1 - Meia1

-Original Message-
From: Cláudio (Prática) [mailto:[EMAIL PROTECTED]
Sent: Monday, March 10, 2003 4:58 PM
To: [EMAIL PROTECTED]
Subject: Re: [obm-l] Problemas


Caro Benedito:

Aqui vai minha solução pro primeiro.

Suponhamos que a aranha tenha n pernas. Seja X(n) o número de maneiras.

Neste caso, cada maneira pode ser representada por uma seqûencia de 16
símbolos distintos:
M(1), M(2), ..., M(n) e S(1), S(2), ..., S(8)
de forma que para cada k (1 = k = n), M(k) sempre preceda S(k).

n = 1:
a única sequencia possível é M(1), S(1) == X(1) = 1

n = k:
para cada sequência correspondente a n = k-1 ( ou seja, 2(k-1) símbolos),
podemos formar uma sequencia correspondnete a n = k, inserindo os símbolos
M(k) e S(k), de forma que M(k) preceda S(k).
Inicialmente, podemos inserir M(k) em 2(k-1) + 1 = 2k - 1 posições
distintas.
Se não houvesse a restrição da precedência, poderíamos inserir S(k) em (2k -
1) + 1 = 2k posições distintas, das quais k teriam M(k) antes de S(k) e k
teriam S(k) antes de M(k).
Descartando estas últimas, ficamos com k posições distintas para S(k).

Logo, temos a recorrência: X(k) = k * (2k - 1) * X(k-1) ==

X(1) = 1
X(2) = 2*3*X(1)
X(3) = 3*5*X(2)
X(4) = 4*7*X(3)
X(5) = 5*9*X(4)
X(6) = 6*11*X(5)
X(7) = 7*13*X(6)
X(8) = 8*15*X(7)

Multiplicando tudo e simplificando, teremos: X(8) = 8! * (15!/(2^7*7!)) =
15! * 8 / 2^7 = 15! / 16.

Um abraço,
Claudio.

- Original Message -
From: benedito [EMAIL PROTECTED]
To: [EMAIL PROTECTED]
Sent: Monday, March 03, 2003 9:31 PM
Subject: [obm-l] Problemas



 Do livro   102 Combinatorial Problems - From the Training of the  USA
IMO
   Team , de Titu Andreescu e Zuming Feng - Birkhäuser. 2003,  dois
  problemas
   interessantes:
  
   1) Uma aranha tem uma meia e um sapato para cada uma de suas  8
pernas. De
   quantas maneiras diferente a aranha pode colocar as meias e os
sapatos,
   supondo  que , em cada perna, a meia tem de ser calçada antes do
sapato?
  
   2) Seja  n =  2^31 . 3^19. Quantos são os divisores inteiros positivos
   de  n^2 que são menores do que  n  mas  não dividem  n?
  
   (Nota:  n^2 =  n elevado a dois)
  
   Benedito Freire


=
Instruções para entrar na lista, sair da lista e usar a lista em
http://www.mat.puc-rio.br/~nicolau/olimp/obm-l.html
O administrador desta lista é [EMAIL PROTECTED]
=
=
Instruções para entrar na lista, sair da lista e usar a lista em
http://www.mat.puc-rio.br/~nicolau/olimp/obm-l.html
O administrador desta lista é [EMAIL PROTECTED]
=


RE: [obm-l] Problema da Tesoura(O Retorno???) e sqrt(pi)

2003-03-31 Por tôpico João Gilberto Ponciano Pereira
É um problema engraçado... Intuitivamente, parece que não dá. Vamos chamar
de perímetro convexo a soma dos arcos convexos de cada pedaço recortado, e
perímetro côncavo a soma dos arcos côncavos de cada pedaço recortado.

A figura inicial tem um perímetro convexo igual a 2pi*r, e um perímetro
côncavo igual a zero. Cada corte em arco, aumenta o perímetro côncavo e o
perímetro convexo na mesma quantidade. Analogamente, cada colagem, reduzem
os perímetros da mesma forma. A figura final tem os dois perímetros igual a
zero...

Tem alguma coisa errada nisso?

-Original Message-
From: Cláudio (Prática) [mailto:[EMAIL PROTECTED]
Sent: Monday, March 31, 2003 5:40 PM
To: [EMAIL PROTECTED]
Subject: Re: [obm-l] Problema da Tesoura(O Retorno???) e sqrt(pi)


Oi, JP:

Eu também já ouvi falar nesse resultado, mas parece que o círculo tem de ser
recortado em 10^50 pedaços, ou algo assim.
De qualquer jeito, se alguém tiver a demonstração, eu gostaria de dar uma
olhada.

Um abraço,
Claudio.

- Original Message -
From: [EMAIL PROTECTED]
To: [EMAIL PROTECTED]
Sent: Monday, March 31, 2003 3:07 PM
Subject: [obm-l] Problema da Tesoura(O Retorno???) e sqrt(pi)


 Turma,alguem sabe demonstrar esse teorema estranho que me apareceu na
Semana
 Olimpica?
 Mostre que e possivel recortar um circulo em varios mas finitos pedaços
 e rearranjar os pedaços sem falhas de modo a formar um quadrado.Cada corte
 deve ser ou um arco de circulo ou um segmento de reta.
 Que tal se esse fosse pra Eureka!?

 TEA WITH ME THAT I BOOK YOUR FACE



=
Instruções para entrar na lista, sair da lista e usar a lista em
http://www.mat.puc-rio.br/~nicolau/olimp/obm-l.html
O administrador desta lista é [EMAIL PROTECTED]
=
=
Instruções para entrar na lista, sair da lista e usar a lista em
http://www.mat.puc-rio.br/~nicolau/olimp/obm-l.html
O administrador desta lista é [EMAIL PROTECTED]
=


RE: [obm-l] Provas da Cone Sul(vamos resolve-las!!!!!)

2003-05-30 Por tôpico João Gilberto Ponciano Pereira
Tá, vai... Imprimi agora e resolvi a primeira que tá babinha
Item a: possível.
Resultados:
a 0 x 1 b
a 1 x 1 c
a 0 x 2 d
b 0 x 2 c
b 2 x 2 d
c 3 x 3 d
 
Item b: impossível.
Os gols sofridos de D (11 gols) está maior do que a soma de todos os gols
marcados de A, B e C juntos(10 gols)!

-Original Message-
From: Johann Peter Gustav Lejeune Dirichlet
[mailto:[EMAIL PROTECTED]
Sent: Thursday, May 29, 2003 1:56 PM
To: [EMAIL PROTECTED]
Subject: [obm-l] Provas da Cone Sul(vamos resolve-las!)


E ai turma,que tal a gente resolver a provba da Cone Sul??

Olimpiada Brasileira de Matematica [EMAIL PROTECTED] wrote: 

Caros(as) amigos(as) da lista,

Ja' estao no nosso arquivo de provas os testes do 1 e 2 dias
da XIV Olimpiada de Matematica do Cone Sul.

http://www.obm.org.br/provas.htm

Abracos, Nelly.

=
Instruções para entrar na lista, sair da lista e usar a lista em
http://www.mat.puc-rio.br/~nicolau/olimp/obm-l.html
=




  _  

Yahoo! Mail  http://br.mail.yahoo.com/ 
Mais espaço, mais segurança e gratuito: caixa postal de 6MB, antivírus,
proteção contra spam.

=
Instruções para entrar na lista, sair da lista e usar a lista em
http://www.mat.puc-rio.br/~nicolau/olimp/obm-l.html
=


RE: [obm-l] Provas da Cone Sul(vamos resolve-las!!!!!)

2003-05-30 Por tôpico João Gilberto Ponciano Pereira
Outra interessante da Cone Sul:
 
Ex. 5:
Seja n = 3k+1, onde k é um inteiro, k=1. Constrói-se um arranjo triangular
de lado n formado por círculos de mesmo raio como o mostrado na figura para
n=7.
Determinar, para cada k, qual o maior número de círculos vermelhos tangentes
entre si.
 
Resposta:
1+3(k-1)(k)/2.
A configuração é simples: para cada linha, coloca-se uma bolinha vermelha e
duas bolinhas brancas. Mais ou menos assim:
 
V
   B B
  B V B
V B B V 
B B V B B 
   B V B B V B
  V B B V B B V
 B B V B B V B B
B V B B V B B V B
   V B B V B B V B B V 
  B B V B B V B B V B B
 B V B B V B B V B B V B
V B B V B B V B B V B B V
 
A prova está no número de tangências de caba bolinha branca com bolinhas
vermelhas na borda (2 no máximo) e no interior (3 no máximo). Vou tentar
formalizar algo mais concreto e envio.

-Original Message-
From: Johann Peter Gustav Lejeune Dirichlet
[mailto:[EMAIL PROTECTED]
Sent: Thursday, May 29, 2003 1:56 PM
To: [EMAIL PROTECTED]
Subject: [obm-l] Provas da Cone Sul(vamos resolve-las!)


E ai turma,que tal a gente resolver a provba da Cone Sul??

Olimpiada Brasileira de Matematica [EMAIL PROTECTED] wrote: 

Caros(as) amigos(as) da lista,

Ja' estao no nosso arquivo de provas os testes do 1 e 2 dias
da XIV Olimpiada de Matematica do Cone Sul.

http://www.obm.org.br/provas.htm

Abracos, Nelly.

=
Instruções para entrar na lista, sair da lista e usar a lista em
http://www.mat.puc-rio.br/~nicolau/olimp/obm-l.html
=




  _  

Yahoo! Mail  http://br.mail.yahoo.com/ 
Mais espaço, mais segurança e gratuito: caixa postal de 6MB, antivírus,
proteção contra spam.

=
Instruções para entrar na lista, sair da lista e usar a lista em
http://www.mat.puc-rio.br/~nicolau/olimp/obm-l.html
=


RE: [obm-l] Problema do torneio das Cidades

2003-06-09 Por tôpico João Gilberto Ponciano Pereira
Não sei se entendi direito o problema, mas acho que dá para resolver assim:
 
Suponha que vc conhece a solução otimizada. Vamos dar uma olhada na borda
inferior do reticulado:
Imagine que nesta borda você tenha a configuração do tipo
/ \. Neste caso, podemos inverter a segunda diagonal de forma a termos a
configuração // sem alterar o número de diagonais do reticulado. O mesmo
raciocínio podemos utilizar para configurações do tipo:
   /
/
podemos descer a diagonal de cima de forma a mais uma vez termos a
configuração //. 
Temos também o caso
   \
/
que invertendo a diagonal de cima, também ficaria //.
 
Fazendo o mesmo para a coluna da esquerda, podemos concluir que existe uma
configuração final otimizada no formato
/
/
/
/
/ / / / / / / / / / /
 
Neste caso, podemos arrancar estas duas linhas e duas colunas e continuar
o problema para um nível inferior (n-2) *(n-2).
 
Logo, seja D(n) o número diagonais para um reticulado de nxn, com n ímpar,
temos que:
D(1) = 1
D(n+2) = D(n) + 2n+3
 
Desenvolvendo, temos que d(n) = (n^2+n)/2
 

-Original Message-
From: Johann Peter Gustav Lejeune Dirichlet
[mailto:[EMAIL PROTECTED]
Sent: Friday, June 06, 2003 4:35 PM
To: [EMAIL PROTECTED]
Subject: [obm-l] Problema do torneio das Cidades


Ola turma!!!Estrou ha dias pensando nesse problema mas nada me ocorreu:
Considere um reticulado n*n, n impar.Nele destacamos alguns segmentos de
comprimento 2^(0,5) ligando dois pontos quaisquer desse reticulado,de modo
que esses segmentinhos nao tenham pontos em comum (nem mesmo extremidades).
Calcule o maximo desses segmentos que podem aparecer.




  _  

Yahoo! Mail  http://br.mail.yahoo.com/ 
Mais espaço, mais segurança e gratuito: caixa postal de 6MB, antivírus,
proteção contra spam.

=
Instruções para entrar na lista, sair da lista e usar a lista em
http://www.mat.puc-rio.br/~nicolau/olimp/obm-l.html
=


[obm-l] RE: [obm-l] Questões da Olimpíada de Maio de 1999 (reenviada)

2003-09-11 Por tôpico João Gilberto Ponciano Pereira
Questão 1:
Um número natural de três algarismos é chamado de tricúbico se é igual a
soma dos cubos dos seus dígitos. Encontre todos os pares de números
consecutivos tais que ambos sejam tricúbicos.
100x + 10y + z = x3 + y3 + z3 
e
100x + 10y + (z+1) = x3 + y3 + (z+1)3
Subtraindo uma da outra e desenvolvendo, temos
z2 + z=0, logo z = 0 (não pode ser negativo)
 
Logo, x3 + y3 é divisível por 10. Analisando os cubos módulo 10 obtemos que
y = x-10
Logo,
100x + 10*(x-10) = x3 + (x-10)^3
x^2 - 13x + 30=0
x = 3 ou x = 10 (não vale)
 
logo, os únicos tricubicos consecutivos são 370 e 371

-Original Message-
From: Rodrigo Maranhão [mailto:[EMAIL PROTECTED]
Sent: Wednesday, September 10, 2003 8:30 PM
To: OBM - Lista
Subject: [obm-l] Questões da Olimpíada de Maio de 1999 (reenviada)



 

Estou reenviando o e-mail pq acho q o Server da lista não o encaminhou já q
estava com figura.




 

Abaixo vão dois problemas da olimpíada de maio de 1999 que eu gostaria de
saber as respostas:

Obs: O problema 1 eu resolvi e achei apenas 1 par de tricúbicos
consecutivos: 370 e 371. No entanto gostaria de confirmar se a resposta é
essa.

 

Problema 1

Um número natural de três algarismos é chamado de tricúbico se é igual a
soma dos cubos dos seus dígitos. Encontre todos os pares de números
consecutivos tais que ambos sejam tricúbicos.

 

Problema 3

A primeira fileira da tabela abaixo se preenche com os números de 1 a 10, em
ordem crescente.

[ ] [ ] [ ] [ ] [ ] [ ] [ ] [ ] [ ] [ ] [ ]

[ ] [ ] [ ] [ ] [ ] [ ] [ ] [ ] [ ] [ ] [ ]

[ ] [ ] [ ] [ ] [ ] [ ] [ ] [ ] [ ] [ ] [ ]

A segunda fileira se preenche com os números de 1 a 10, em qualquer ordem.

Em cada casa da terceira fileira se escreve a soma dos dois números escritos
nas casas acima.

Existe alguma maneira de preencher a segunda fileira de modo que os
algarismos das unidades dos números da terceira fileira sejam todos
distintos?

 

=
Instruções para entrar na lista, sair da lista e usar a lista em
http://www.mat.puc-rio.br/~nicolau/olimp/obm-l.html
=


RE: [obm-l] Torneio de tenis

2003-09-22 Por tôpico João Gilberto Ponciano Pereira
Acho que 2^k não abrange todas as possibilidades de conjunto. Consegui uma
configuração válida para n=12.
 
Vamos imaginar uma matriz nxn, onde DIA(A,B) é o dia do jogo do jogador A
versus o jogador B. Consideremos DIA(A,A) = 0, pois não faz sentido o
jogador A jogar contra si mesmo. Por definição, DIA(A,B) = DIA(B,A).
 
Um resultado possível para o problema com 12 jogadores seria:
0 1 2 3 4 5 6 7 8 9 10 11
1 0 3 4 5 6 7 8 9 10 11 2
2 3 0 5 6 7 8 9 10 11 4 1
3 4 5 0 7 8 9 10 11 2 1 6
4 5 6 7 0 9 10 11 2 1 8 3
5 6 7 8 9 0 11 2 1 4 3 10
6 7 8 9 10 11 0 1 4 3 2 5
7 8 9 10 11 2 1 0 3 6 5 4
8 9 10 11 2 1 4 3 0 5 6 7
9 10 11 2 1 4 3 6 5 0 7 8
10 11 4 1 8 3 2 5 6 7 0 9
11 2 1 6 3 10 5 4 7 8 9 0
 
Note que a soma de cada linha e cada coluna deve ter o mesmo valor, que é
igual a n * (n-1)/2.
 
Meu palpite é que a soma (n*(n-1)/2) deve ser par, ou seja, n = 4*k, além da
configuração n=2. Acho que dá para provar que é possível construir esta
matriz:
1 - a primeira linha / coluna com os valores de 0 a n
2 - a diagonal obviamente terá zeros
3 - os dias devem ser intercalados em par/impar. Por exemplo, se DIA(A,B) é
par, DIA(A,B+1) deve ser obrigatoriamente impar
4 - a matriz é preenchida sempre com o menor número, desde que se satisfaçam
as condições anteriores.
 
Mas aí eu lhe pergunto: E para provar isso tudo, hein??

-Original Message-
From: Domingos Jr. [mailto:[EMAIL PROTECTED]
Sent: Monday, September 22, 2003 5:44 PM
To: [EMAIL PROTECTED]
Subject: Re: [obm-l] Torneio de tenis


Estou com um palpite: n = 2^k
 
A idéia é simples:
 
Sejam A e B conjuntos de tenistas com |A| = |B| = k, então são necessários
no mínimo k dias para que todos os tenistas de A enfrentem todos de B sendo
que todo jogador joga uma vez por dia nesses k dias, por exemplo:
A = {x1, x2, ..., xk}, B = {y1, y2, ..., yk}
dia 1: (x1, y1), (x2, y2), , (xk, yk)
dia 2: (x1, y2), (x2, y3), , (x[k-1], yk), (xk, y1)
dia 3: (x1, y3), (x2, y4), , (x[k-2], yk), (x[k-1], y1), (xk, y2)
...
dia k: (x1, yk), (x2, y1), ..., (xk, y[k-1])
 
Agora note que no caso |T| = n = 2^k podemos fazer todos se enfrentarem em n
- 1 dias:
divida T em T1 e T2 (|T1| = n/2 = |T2|) e faça todos de T1 enfrentarem todos
de T2 em n2 = 2^(k-1) dias.
agora todos os tenistas em T1 precisam se enfrentar entre si, e todos de T2
precisam se enfrentar entre si também, como os conjuntos são disjuntos, os
processos (jogos dentro de cada partição) podem ocorrer em paralelo, logo
temos que o total de dias necessários (mínimos) é:
2^(k-1) + 2^(k-2) +  + 2 + 1 = 2^k - 1 = n - 1 dias!
 
não estou tendo idéias de como provar essa minha conjectura, não dá pra ser
simplista e assumir que a maneira ótima de organizar os jogos é
particionando os tenistas em dois conjuntos iguais!
 
vou pensar melhor!
 
[ ]'s

- Original Message - 
From: Johann Peter Gustav Lejeune  mailto:[EMAIL PROTECTED]
Dirichlet 
To: [EMAIL PROTECTED] mailto:[EMAIL PROTECTED]  
Sent: Monday, September 22, 2003 1:49 PM
Subject: [obm-l] Torneio de tenis

Oi turma, estou tentando resolver esse problema pra fechar a soluçao de um
problema da IMO:
Considere n inteiro positivo, e um torneio de tenis no qual todos os n
jogadores jogam contra todos.
Sabe-se que e possivel distribuir as partidas em n-1 dias de modo que cada
jogador jogue exatamente uma vez por dia.
Ache todos os possiveis valores de n.




  _  

Desafio  http://br.rd.yahoo.com/s/c/m/?http://cade.com.br/antizona
AntiZona: participe do jogo de perguntas e respostas que vai dar
1 Renault Clio, computadores, câmeras digitais, videogames e muito mais!

=
Instruções para entrar na lista, sair da lista e usar a lista em
http://www.mat.puc-rio.br/~nicolau/olimp/obm-l.html
=


RE: [obm-l] Re: N/A

2003-11-04 Por tôpico João Gilberto Ponciano Pereira

Pessoal

Sem querer ser chato, mas cheguei ao resultado de 55. O processo é um pouco
feio, mas chega lá.

De quantas maneiras podemos formar uma sequencia de oito bits(0 ou 1) de
forma que nunca  apareça nesta sequencia zeros adjacentes

Seja A(n) o número de combinações dentro das regras que terminam com o bit
1, e B(n) os que terminem com o bit 0.

É fácil ver que:
1) A(n+1) = A(n) + B(n)
2) B(n+1) = A(n)

Logo:
A(n+1) + B(n+1) = 2*A(n) + B(n)
e substituindo, temos:
A(n+2) = 2*A(n) + A(n-1)

Sabendo que:
 A(1) = 1 == (1)
 A(2) = 2 == (01, 11)
 A(3) = 3 == (011, 101, 111) obs: 001 não vale!
podemos seguir com a recorrência até A(9) = A(8) + B(8) = 55

Um abraço!
JG

-Original Message-
From: Augusto Cesar de Oliveira Morgado [mailto:[EMAIL PROTECTED]
Sent: Monday, November 03, 2003 9:29 PM
To: [EMAIL PROTECTED]
Subject: Re: [obm-l] Re: N/A


Recebi a mensagem que enviei com um rosto amarelo com cara de idiota
sorrindo 
no lugar em que digitei o numero 8. Desculpas a todos.
Morgado


--
CIP WebMAIL - Nova Geração - v. 2.1
CentroIn Internet Providerhttp://www.centroin.com.br
Tel: (21) 2542-4849, (21) 2295-3331  Fax: (21) 2295-2978
Empresa 100% Brasileira - Desde 1992


-- Original Message ---
From: Augusto Cesar de Oliveira Morgado [EMAIL PROTECTED]
To: [EMAIL PROTECTED]
Sent: Mon, 3 Nov 2003 20:59:29 -0200
Subject: [obm-l] Re: N/A

 Seja f(n) a resposta para uma sequencia de n bits. Ou a seq. começa 
 em 1 ou começa em 01. Logo, f(n)=f(n-1)+f(n-2). Como f(1) = 2 e f(2) 
 = 3, f(3) = 2+3=5, f(4) = 5+3 = 8, f(5) = 8+5 = 13, f(6)=13=8 = 21,
  f(7) = 21+13 = 44 e f(8) = 44+21 = 65.
 
 --
 CIP WebMAIL - Nova Geração - v. 2.1
 CentroIn Internet Providerhttp://www.centroin.com.br
 Tel: (21) 2542-4849, (21) 2295-3331  Fax: (21) 2295-2978
 Empresa 100% Brasileira - Desde 1992
 
 -- Original Message ---
 From: Daniel Faria [EMAIL PROTECTED]
 To: [EMAIL PROTECTED]
 Sent: Mon, 03 Nov 2003 19:16:55 -0200
 Subject: N/A
 
  Ainda nao consegui finalizar este exercício:
  
  De quantas maneiras podemos formar uma sequencia de oito bits(0 ou 1)
   de forma que nunca  apareça nesta sequencia zeros adjacentes ( _ _ 
  0 0 _ _ _ _ ).
  
  Obrigado.
  
  _
  MSN Hotmail, o maior webmail do Brasil.  http://www.hotmail.com
  
 
=
  Instruções para entrar na lista, sair da lista e usar a lista em
  http://www.mat.puc-rio.br/~nicolau/olimp/obm-l.html
 
=
 --- End of Original Message ---
 
 =
 Instruções para entrar na lista, sair da lista e usar a lista em
 http://www.mat.puc-rio.br/~nicolau/olimp/obm-l.html
 =
--- End of Original Message ---

=
Instruções para entrar na lista, sair da lista e usar a lista em
http://www.mat.puc-rio.br/~nicolau/olimp/obm-l.html
=
=
Instruções para entrar na lista, sair da lista e usar a lista em
http://www.mat.puc-rio.br/~nicolau/olimp/obm-l.html
=


RE: [obm-l] Prova do IME

2003-11-05 Por tôpico João Gilberto Ponciano Pereira
Já que é assim, para a segunda:
P(x) = x3 + ax + b e b0 e P(x) possui 3 raízes reais, prove que a0

Se P(x) possui 3 raízes reais, P(x) não é estritamente crescente ou
estritamente decrescente. Logo, P'(x) terá 2 raízes reais.

P'(x) = 3x2 + a, com raízes x = (-a/3) ^.5, logo a =0.

Entretanto, a=0 implica em P(x) sempre crescente com exceção do ponto x=0, e
como x=0 não é um ponto de raiz tripla (b0), podemos concluir que a0.

Muito feio?

-Original Message-
From: Paulo Santa Rita [mailto:[EMAIL PROTECTED]
Sent: Wednesday, November 05, 2003 1:05 PM
To: [EMAIL PROTECTED]
Subject: Re: [obm-l] Prova do IME


Ola Prof Morgado e demais
colegas desta lista ... OBM-L,

Deu uma olhada no site do GPI. A prova esta la, questao por questao. Mas ...

FEITA ! Que pena, nao vamos ter a alegria de descobrir as solucoes. Mas eu 
proponho o seguinte :

Vamos encontrar, pra cada questao, uma maneira diferente de fazer (  mesmo 
que seja mais feia ou longa ) ? Eu começo :

1 QUESTAO ) Existe uma regra, chamada regra de Chio, que permite abaixar a 
ordem de uma matriz. Basta que A11=1 ( se nao me falha a memoria ). Entao, 
aplicando a regra de Chio duas vezes vamos cair numa matriz 2x2, cujo 
determinante e facil calcular.

Calculando o determinante em funcao de N, igualamos a 5 e resolvemos a 
equacao.

Quem faz a questao 2, com solucao diferente da do GPI ? Para que todos 
possam participar, voces aceitam que uma pessoa so possa fazer uma questao (

nao duas ou mais ) ?

Um Abraco a Todos !
Paulo Santa Rita
4,1403,051103





From: Augusto Cesar de Oliveira Morgado [EMAIL PROTECTED]
Reply-To: [EMAIL PROTECTED]
To: [EMAIL PROTECTED]
Subject: Re: [obm-l] Prova do IME
Date: Wed, 5 Nov 2003 14:22:54 -0200
MIME-Version: 1.0
Received: from mc2-f14.hotmail.com ([65.54.237.21]) by mc2-s17.hotmail.com 
with Microsoft SMTPSVC(5.0.2195.5600); Wed, 5 Nov 2003 07:25:29 -0800
Received: from sucuri.mat.puc-rio.br ([139.82.27.7]) by mc2-f14.hotmail.com

with Microsoft SMTPSVC(5.0.2195.5600); Wed, 5 Nov 2003 07:24:22 -0800
Received: (from [EMAIL PROTECTED])by sucuri.mat.puc-rio.br (8.9.3/8.9.3) 
id NAA07638for obm-l-MTTP; Wed, 5 Nov 2003 13:23:26 -0200
Received: from gorgo.centroin.com.br (gorgo.centroin.com.br 
[200.225.63.128])by sucuri.mat.puc-rio.br (8.9.3/8.9.3) with ESMTP id 
NAA07634for [EMAIL PROTECTED]; Wed, 5 Nov 2003 13:23:25 -0200
Received: from centroin.com.br (trex.centroin.com.br [200.225.63.134])by 
gorgo.centroin.com.br (8.12.10/8.12.9) with ESMTP id hA5FMtad007873for 
[EMAIL PROTECTED]; Wed, 5 Nov 2003 13:22:55 -0200 (EDT)
X-Message-Info: HQbIehuYceSUWy5LlRlpF6fIONaJJTv2iApCFDJ/N8U=
Message-Id: [EMAIL PROTECTED]
In-Reply-To: [EMAIL PROTECTED]
References: [EMAIL PROTECTED]
X-Mailer: CIP WebMail 2.10 experimantal 20030731a
X-OriginatingIP: 200.141.90.78 (morgado)
Sender: [EMAIL PROTECTED]
Precedence: bulk
Return-Path: [EMAIL PROTECTED]
X-OriginalArrivalTime: 05 Nov 2003 15:24:23.0275 (UTC) 
FILETIME=[E41663B0:01C3A3B0]

www.gpi.g12.br


--
CIP WebMAIL - Nova Geração - v. 2.1
CentroIn Internet Providerhttp://www.centroin.com.br
Tel: (21) 2542-4849, (21) 2295-3331  Fax: (21) 2295-2978
Empresa 100% Brasileira - Desde 1992


-- Original Message ---
From: Paulo Santa Rita [EMAIL PROTECTED]
To: [EMAIL PROTECTED]
Sent: Wed, 05 Nov 2003 13:20:15 +
Subject: [obm-l] Prova do IME

  Ola Pessoal !
 
  Todos devem ter notado que novamente estamos sendo agredidos por um
  imbecil qualquer. Nao se responde a este tipo de gente. Vamos
  ignorar as mensagens ofensivas e trata-las como sao : lixo produzido
  por lixo.
 
  Se nao me engano, ontem foi a prova de Matematica do IME. Alguem tem
  a prova ou sabe onde encontra-la na Internet ? Seria interessante
  disponibiliza-la aqui na nossa lista, para que pudessemos discutir
  algumas questoes.
 
  Um Abraco a Todos !
  Paulo Santa Rita
  4,1120,051103
 
  _
  MSN Hotmail, o maior webmail do Brasil.  http://www.hotmail.com
 
  
=
  Instruções para entrar na lista, sair da lista e usar a lista em
  http://www.mat.puc-rio.br/~nicolau/olimp/obm-l.html
  
=
--- End of Original Message ---

=
Instruções para entrar na lista, sair da lista e usar a lista em
http://www.mat.puc-rio.br/~nicolau/olimp/obm-l.html
=

_
MSN Messenger: converse com os seus amigos online.  
http://messenger.msn.com.br

=
Instruções para entrar na lista, sair da lista e usar a lista em
http://www.mat.puc-rio.br/~nicolau/olimp/obm-l.html

RE: [obm-l] Problema do Camelo - solucao

2003-11-20 Por tôpico João Gilberto Ponciano Pereira
Olá, pessoal

Bem, Rogério, eu, como o prof. Nicolau, também tenho minhas dúvidas se ficou
provado que esta é a solução ótima. Acho que o principal motivo para isto é
o simples fato que no primeiro trecho, o dos 3 nanômetros, a última viagem é
feita sem que o camelo esteja 100% carregado. Ou seja, existe uma folga
onde podemos imaginar algumas soluções alternativas. (Se numa viagem, o
camelo vai com menos que 100% de carga, é possível provar que todas as
viagens daquele trecho podem ir com menos de 100% de carga).

Outra coisa que não fico confortável é com o fato de usarmos apenas N+1
viagens para 100 * N litros. Fiz algumas contas, e a degradação no
rendimento entre fazer N+1 e N+2 viagens é pequena, se formos considerar o
ganho em distância. Acho que, principalmente nos últimos trechos, podemos
jogar com estes números, de forma a conseguirmos distâncias finais mais
próximas aos exatos 10km.



-Original Message-
From: Rogerio Ponce [mailto:[EMAIL PROTECTED]
Sent: Wednesday, November 19, 2003 7:24 PM
To: [EMAIL PROTECTED]
Subject: Re: [obm-l] Problema do Camelo - solucao 


Não gostei , e alterei associado a este trecho por associado a este 
último trecho :

Olá Nicolau,
repare que partimos de uma condição de contorno , que era ter 1000L
no final.

O mínimo para isso , seriam 11 viagens de ida a partir da última
base . Temos que adotar isso, pois só desperdiçaríamos água se
aumentássemos o número de viagens para transportar a mesma
quantidade de água entre a ultima base e o ponto final.

Ao escolhermos que as 11 partidas seriam com tanque cheio (100L) ,
estamos minimizando o caminho que falta percorrer do ponto inicial
até essa última base , ao mesmo tempo em que também minimizamos o
custo do transporte da água associado a este último trecho do caminho .

O mesmo raciocínio se aplica sucessivamente a todos os trechos.
[]´s
Rogério.



From: Nicolau C. Saldanha [EMAIL PROTECTED]
...

Mas também não demonstrou que a resposta é mínima, pelo menos não de forma
clara e explícita.

[]s, N.

_
MSN Hotmail, o maior webmail do Brasil.  http://www.hotmail.com

=
Instruções para entrar na lista, sair da lista e usar a lista em
http://www.mat.puc-rio.br/~nicolau/olimp/obm-l.html
=
=
Instruções para entrar na lista, sair da lista e usar a lista em
http://www.mat.puc-rio.br/~nicolau/olimp/obm-l.html
=


RE: [obm-l] Probabilidade 1/3

2003-11-20 Por tôpico João Gilberto Ponciano Pereira
Não sei não, hein? Intuitivamente, acho que uma altura de ~ 1/3 do diâmetro
não seria a configuração que daria chances iguais para todas as faces.

Tinha pensado assim: Imaginem que o cilindro tocou o plano. Neste momento, o
eixo está inclinado num ângulo Â, que pode variar de 0 a 360. Eu diria que
poderíamos dividir os 360 graus em 3 conjuntos, de acordo com a face final
que o dado apresentaria. Exemplificando:

CARA: -60 a 60 graus
FACE NOVA: de 60 a 120 graus União com 240 a 300 graus
COROA: 120 a 240 graus.

A idéia é que a soma das aberturas angulares de cada conjunto seja igual, ou
seja, 120 graus para cada abertura. O ponto de inflexão, ou seja, o ângulo
com que o cilindro muda de face quando cai completamente, ocorreria quando a
reta normal, que sai do centro de massa do cilindro, encontra o ponto de
contato. Ou seja, se a reta normal está à esquerda do ponto de contato, o
cilindro irá rodar para aquele lado, e vice-versa.

Neste caso, pelas minhas contas, H=d*tg30, algo que eu considero um pouco
mais intuitivamente correto. O que acham?

-Original Message-
From: Claudio Buffara [mailto:[EMAIL PROTECTED]
Sent: Wednesday, November 19, 2003 11:34 PM
To: [EMAIL PROTECTED]
Subject: Re: [obm-l] Probabilidade 1/3


on 19.11.03 21:41, Nicolau C. Saldanha at [EMAIL PROTECTED]
wrote:

 Podemos simplificar um pouco escolhendo aleatoriamente um vetor na esfera
S^2
 centrada no centro de massa e vendo onde vai parar o prolongamento do
vetor.
 
 No caso do cilindro a esfera fica dividida em três partes, duas calotas
 polares correspondentes às duas faces e uma região tropical correspondente
 à superfície cilíndrica. As áreas destas regiões são proporcionais às
alturas.
 Podemos tomar a esfera de diâmetro sqrt(h^2+d^2) e assim a região tropical
 tem probabilidade h/sqrt(h^2+d^2). Para que isto seja igual a 1/3 devemos
ter
 sqrt(h^2+d^2) = 3h ou h^2+d^2 = 9h^2 ou h = d/sqrt(8).
 
Concordo! Eu havia olhado apenas pra secao transversal do cilindro e nao pro
solido inteiro.

 Mais este modelo me parece muito simplista... sei lá...
 
Tambem concordo, mas acho que a sua solucao eh a correta se supusermos que o
choque da moeda com a superficie sobre a qual ela eh lancada for
perfeitamente inelastico.

Um abraco,
Claudio.

=
Instruções para entrar na lista, sair da lista e usar a lista em
http://www.mat.puc-rio.br/~nicolau/olimp/obm-l.html
=
=
Instruções para entrar na lista, sair da lista e usar a lista em
http://www.mat.puc-rio.br/~nicolau/olimp/obm-l.html
=


RE: [obm-l] Divisao de um Quadrilatero

2004-03-04 Por tôpico João Gilberto Ponciano Pereira
Olha, não sei se burocrática seria a palavra correta, mas eu pensei em fazer
assim:

Seja o quadrilátero ABCD, de forma que o Área(ACD)  Área(ABC)
Traçamos a reta r paralela a AC passando por B. Seja o ponto E a intersecção
de r com o prolongamento do lado AD. É fácil ver que Área(ABC)=Área(AEC),
logo, Área(ABCD) = Área(EDC). Logo, a reta que divide o quadrilátero em duas
figuras de áreas iguais seria a mediatriz do triângulo EDC passando por C.

Aliás, por um raciocínio semelhante, conseguimos chegar a uma resposta
genérica para um eneágono convexo, é só ter paciência de projetar todos os
triângulo formados com o vértice escolhido...


-Original Message-
From: Claudio Buffara [mailto:[EMAIL PROTECTED]
Sent: Thursday, March 04, 2004 1:50 PM
To: [EMAIL PROTECTED]
Subject: Re: [obm-l] Divisao de um Quadrilatero


Acabei de resolver esse dai. Eh menos interessante do que eu imaginava (pelo
menos a minha solucao eh bem burocratica). Mesmo assim, se alguem tiver uma
solucao inteligente, por favor envie pra lista.

[]'s,
Claudio.

on 04.03.04 12:20, Claudio Buffara at [EMAIL PROTECTED] wrote:

 Esqueci de dizer uma coisa: eu nao consegui resolver este problema e,
 portanto, qualquer ajuda serah bem vinda.
 
 []'s,
 Claudio.
 
 on 04.03.04 10:31, Claudio Buffara at [EMAIL PROTECTED] wrote:
 
 Oi, pessoal:
 
 Aqui vai um de geometria:
 
 Dado um quadrilatero convexo, mostre como construir uma reta que bissecte
a
 sua area usando apenas regua e compasso.
 
 O problema pode ser genertalizada para um n-gono convexo.
 
 Um abraco,
 Claudio.
 
 

=
Instruções para entrar na lista, sair da lista e usar a lista em
http://www.mat.puc-rio.br/~nicolau/olimp/obm-l.html
=

=
Instruções para entrar na lista, sair da lista e usar a lista em
http://www.mat.puc-rio.br/~nicolau/olimp/obm-l.html
=


RE: [obm-l] Problema estranho..

2004-03-23 Por tôpico João Gilberto Ponciano Pereira
Se vc considerar o 0 como natural, dá para fazer assim:
 
0 8 4
7 3 2
5 1 6
 
Mas o legal mesmo é fazer este exercício para uma matriz 20x20. Quem
advinha?

-Original Message-
From: Cloves Jr [mailto:[EMAIL PROTECTED]
Sent: Tuesday, March 23, 2004 4:37 PM
To: Grupo OBM
Subject: [obm-l] Problema estranho..


Olá pessoal, eu normalmente naum ajudo muito nas discussoes por estar ainda
no primeiro ano da facu mas estou precisando da ajuda de vcs...
 
Eu sei que eh um problema basico mas eu naum consegui resolver:
 
Dada uma matriz 3x3, encontrar os coeficientes tal que a soma de cada linha
e cada coluna seja 12. Os coeficientes não podem ser repetidos e todos são
naturais.
 
Eu acho que eh impossivel mas se alguem conseguir resolver por favor
gostaria de saber como...
 
[]s
 
Cloves Jr

ICQ: 148686592   http://web.icq.com/whitepages/online?icq=148686592img=21


 


=
Instruções para entrar na lista, sair da lista e usar a lista em
http://www.mat.puc-rio.br/~nicolau/olimp/obm-l.html
=


RE: [obm-l] Problema estranho..

2004-03-23 Por tôpico João Gilberto Ponciano Pereira
Aliás, é um problema interessante. Eu sei que para qualque matriz N x N  com
n ímpar é possível montar a tal da matriz. A formulação tradicional é para
os Naturais SEM o zero, e a soma das linhas ou colunas seria sempre igual a
N * ( N^2 + 1) / 2. (No caso da 3x3, a soma seria 15)

Agora, quando N é par...

-Original Message-
From: Artur Costa Steiner [mailto:[EMAIL PROTECTED]
Sent: Tuesday, March 23, 2004 6:03 PM
To: [EMAIL PROTECTED]
Subject: RE: [obm-l] Problema estranho..


Eh, aih dah. Mas se vc seguir a convencao usual de que o 0 nao eh natural,
entao o problema eh impossivel.
Artur

- Mensagem Original 
De: [EMAIL PROTECTED]
Para: '[EMAIL PROTECTED]' [EMAIL PROTECTED]
Assunto: RE: [obm-l] Problema estranho..
Data: 23/03/04 23:53

Se vc considerar o 0 como natural, dá para fazer assim:

0 8 4
7 3 2
5 1 6

Mas o legal mesmo é fazer este exercício para uma matriz 20x20. Quem
advinha?

-Original Message-
From: Cloves Jr [mailto:[EMAIL PROTECTED]
Sent: Tuesday, March 23, 2004 4:37 PM
To: Grupo OBM
Subject: [obm-l] Problema estranho..


Olá pessoal, eu normalmente naum ajudo muito nas discussoes por estar ainda
no primeiro ano da facu mas estou precisando da ajuda de vcs...

Eu sei que eh um problema basico mas eu naum consegui resolver:

Dada uma matriz 3x3, encontrar os coeficientes tal que a soma de cada linha
e cada coluna seja 12. Os coeficientes não podem ser repetidos e todos são
naturais.

Eu acho que eh impossivel mas se alguem conseguir resolver por favor
gostaria de saber como...

[]s

Cloves Jr

ICQ: 148686592 http://web.icq.com/whitepages/online?icq=148686592img=21





=
Instruções para entrar na lista, sair da lista e usar a lista em
http://www.mat.puc-rio.br/~nicolau/olimp/obm-l.html
=


OPEN Internet
@ Primeiro provedor do DF com anti-vírus no servidor de e-mails @


=
Instruções para entrar na lista, sair da lista e usar a lista em
http://www.mat.puc-rio.br/~nicolau/olimp/obm-l.html
=

=
Instruções para entrar na lista, sair da lista e usar a lista em
http://www.mat.puc-rio.br/~nicolau/olimp/obm-l.html
=


[obm-l] RE: [obm-l] Mais casas de pombos (uma idéia)

2004-05-11 Por tôpico João Gilberto Ponciano Pereira
Tenho uma idéia...
Vamos pensar pequeno primeiro, um conjunto de 3 números (A, B, c), tal que
A+B  A+C  B+C. Um exemplo seria o conjunto (1, 2, 4), e o número de
somas diferentes seria o binomial (3,2).
Para 4, um exemplo seria (1,2,4,7), e o número de somas seria o binomial
(4,2), e a soma máxima seria 4+7 = 11.
 
Ora, se formos considerar um conjunto com N elementos, e nenhum deles é
maior que 2004, podemos concluir que a soma máxima de dois elementos de M é
4005, logo, dado um conjunto N, podemos ter 4005 combinações diferentes.
 
Fazendo as contas, já podemos concluir que N = 90.
Mas isso ainda não prova nada, é só um limitante. Acho que o Cláudio pensou
em algo parecido.

-Original Message-
From: Johann Peter Gustav Lejeune Dirichlet
[mailto:[EMAIL PROTECTED]
Sent: Tuesday, May 11, 2004 5:00 PM
To: [EMAIL PROTECTED]
Subject: Re: [obm-l] Mais casas de pombos (uma ideia)


Ola Claudio!
estou tentando analisar casos pequenos nesse problema.
Minha ideia e tentar escrever isto com linguagem de grafos. O problema e que
eu nao sei como observar hipergrafos :(
Outra ideia e calcular quantas somas de dois elementos existem e que sao
diferentes. E muita conta mas vale a pena...
PS.: Passa a sua demo pra gente tentar melhorar...

Claudio Buffara [EMAIL PROTECTED] wrote:

Oi, Fred (e demais colegas):

Jah que estamos nesse assunto, aqui vai um problema que ainda estah em
aberto na lista:

Ache o menor inteiro N tal que dados quaisquer N elementos distintos do
conjunto {1,2,3,...,2004}, existem 4 elementos distintos dentre os N tais
que a soma de dois deles eh igual a soma dos outros dois.

Por enquanto, eu soh consegui provar que o N critico eh = 90.

[]s,
Claudio.

=
Instruções para entrar na lista, sair da lista e usar a lista em
http://www.mat.puc-rio.br/~nicolau/olimp/obm-l.html
=r/~
nicolau/olimp/obm-l.html
=



TRANSIRE SVVM PECTVS MVNDOQVE POTIRI 

CONGREGATI EX TOTO ORBE MATHEMATICI OB SCRIPTA INSIGNIA TRIBVERE 

Fields Medal(John Charles Fields)
 
N.F.C. (Ne Fronti Crede)




  _  

 
http://br.rd.yahoo.com//mail_br/tagline/?http://br.download.yahoo.com/messe
nger/ Yahoo! Messenger - Fale com seus amigos online. Instale
http://br.rd.yahoo.com//mail_br/tagline/?http://br.download.yahoo.com/messe
nger/ agora!


=
Instruções para entrar na lista, sair da lista e usar a lista em
http://www.mat.puc-rio.br/~nicolau/olimp/obm-l.html
=


[obm-l] Jogos de Azar

2004-05-17 Por tôpico João Gilberto Ponciano Pereira
Pessoal
 
Estou com uma dúvida que anda me perturbando sobre o jogo do bicho. Sei que
é um jogo de azar, onde no final, quem sai ganhando é sempre a banca, mas me
fizeram a seguinte proposição:
Na aposta mais básica, a chance é de 1 em 25 bichos, ou seja, 4%. Neste
caso, o ganho é de 15 vezes o valor da aposta. 
Suponhamos agora uma série de apostas, onda a inicial (AI) é de R$1,00, e a
dos dias seguintes em função de um fator multiplicador (FM) de 1.1, tal que
Aposta do dia A(d) = AI * FM ^ d
 
O custo total envolvido até um determinado dia é a soma dos termos desta PG,
ou seja, A(0) + A(1) + A(2) + ... + A(N) = AI * (1- FM^N) / (1-FM), mas caso
seja premiado, o ganho será sempre de AI * FM ^ N, maior que o valor da soma
das apostas até o momento.  Ou seja, se jogar todos os dias, supondo que um
dia eu realmente chegue a ganhar (afinal, 4% não é uma chance assim tão
pequena), seguindo este método, sempre sairei no lucro.
 
Não parece que algo está errado nesta conta toda? Alguém além de mim ficou
tentado a jogar também?

=
Instruções para entrar na lista, sair da lista e usar a lista em
http://www.mat.puc-rio.br/~nicolau/olimp/obm-l.html
=


RE: [obm-l] Jogos de Azar

2004-05-17 Por tôpico João Gilberto Ponciano Pereira
Tudo bem, tudo bem, a grana é alta, mas tem algumas coisas que temos que
reavaliar

Na centésima aposta, o valor do jogo seria de R$1,00 * 1.1 ^99 = R$
12.527,83, e o prêmio seria de 15 vezes isso, ou seja, R$ 187.917,45, mas
como já gastamos R$ 137.796,12 , o lucro seria não de míseros R$600,00, mas
sim de mais de R$ 50.000,00 reais! Cerca de 36% do capital investido. (qual
aplicação fornece esta taxa de retorno?) Além disso, a chance de chegarmos
na centésima aposta e ainda não ter ganho é de ~ 1.7% (1 - 0.96 ^100)

Tudo bem, usando N = 1.08, chegamos a valores mais modestos... Para a 100a
tentativa, o valor total seria de ~ 27k, e o lucro de ~30k, cerca de 10% de
lucro

Ah, e segundo minhas fontes (não muito confiáveis), o jogo do bicho sorteia
3 números por dia, ou seja, tudo isso pode acontecer em apenas 1 mês. De
qualquer forma, podemos usar o mesmo raciocínio para um jogo de roleta em um
cassino, por exemplo. A chance de ganhar é menor, 1/37, mas a proporção do
pagamento é maior...

-Original Message-
From: Domingos Jr. [mailto:[EMAIL PROTECTED]
Sent: Monday, May 17, 2004 2:51 PM
To: [EMAIL PROTECTED]
Subject: Re: [obm-l] Jogos de Azar


Isso funciona, mas note que o valor da aposta está crescendo
exponencialmente... ou seja, pode acontecer de você ficar sem dinheiro antes
de conseguir vencer...

Eu calculei o valor esperado de ganho quando você faz uma aposta de 1 real
no começo e aumenta por um fator 1.1 cada aposta até ganhar ou após perder
100 vezes.
O valor é 671,212 reais... a pergunta agora é: vale a pena?

Se você tem muito dinheiro (o suficiente para garantir até 100 apostas é
mais de 137.000 reais) esses 671,212 reais esperados não valem a pena pois
não rendem nem os juros bancários...
Se você tem pouco, vai quebrar antes da banca com boa probabilidade ou vai
ganhar uns trocados apenas.

[ ]'s

=
Instruções para entrar na lista, sair da lista e usar a lista em
http://www.mat.puc-rio.br/~nicolau/olimp/obm-l.html
=

=
Instruções para entrar na lista, sair da lista e usar a lista em
http://www.mat.puc-rio.br/~nicolau/olimp/obm-l.html
=


[obm-l] RE: [obm-l] 1º ex. cone sul 2004

2004-05-28 Por tôpico João Gilberto Ponciano Pereira
Só coloquei mais uma ressalva quando resolvi... A soma dos algarismos não
pode ser divisível por 3. Você consegue eliminar mais duas possibilidades
sem fazer contas absurdas. De qualquer forma, ainda acho o exercício muito
braçal por esta solução. Alguém viu coisa melhor?

-Original Message-
From: Osvaldo [mailto:[EMAIL PROTECTED]
Sent: Friday, May 28, 2004 3:46 AM
To: obm-l
Subject: [obm-l] 1º ex. cone sul 2004


Pessoal tava tentando resolver o prim. prob. e fiz um 
esboço, gostaria que me ajudassem a termina lo.

Sejam x, y e z os algarismos escolhidos por Maxi.
Formam-se 6 números de três algarismos, logo x, y e z 
são não nulos.
Fazendo a análise de todos os quadrados perfeitos de 
três alg. que não contenham nenhum dígito nulo temos 19 
possibilidades : {121, 144, 169, 196, 225, 256, 289, 
324, 361, 441, 484, 529, 576, 625, 676, 729, 784, 841, 
961}=P


Na verdade 16 pois 144 é um núm. obtido pela 
reordenação dos dígitos de 441 (ou vice-versa); o mesmo 
ocorre com 169, 196 e 961.

Observe agora que como tres dos números formadas são 
primos então devemos ter, necessariamente, que o dígito 
das unidades destes números primos sejam ímpares.

Assim, das 17 possibilidades extraimos somente 5 
possíveis uma vez que todos os números que possuam 2 
alg. pares formam 5 números (atraves da reordenação dos 
dígitos destes) pares, logo esses números não contêm os 
dígitos que Maxi escolheu e o nº 121 não forma 6 
reodenaçoes distintas.


As 5 possibilidades são:
169, 361, 529, 576, 729


Observe que todos têm um nº par logo uma reordenaçao de 
cada um dos n° nao sera um n° primo. Cada um dos 6 
numeros tambem nao sao primos. Assim resta-se 
inpecionar 3 ou 4 reordenaçoes de cada um dos 5 nº.

Para o nº 169 a reodenação 961=31^2 é composto, logo 
169 nao forma 3 nºs primos.
Assim resta-me analisar:

3,6,1
5,2,9
5,7,6
7,2,9

Alguém poderia encontrar uma maneira menos braçal de 
inspecionar a 'primalidade' dos numeros formados pela 
reordenaçao dos digitos acima?



Atenciosamente,

Engenharia Elétrica - UNESP Ilha Solteira
Osvaldo Mello Sponquiado 
Usuário de GNU/Linux


 
__
Acabe com aquelas janelinhas que pulam na sua tela.
AntiPop-up UOL - É grátis!
http://antipopup.uol.com.br/



=
Instruções para entrar na lista, sair da lista e usar a lista em
http://www.mat.puc-rio.br/~nicolau/olimp/obm-l.html
=

=
Instruções para entrar na lista, sair da lista e usar a lista em
http://www.mat.puc-rio.br/~nicolau/olimp/obm-l.html
=


[obm-l] RE: [obm-l] Cosseno não é Polinômio

2004-05-31 Por tôpico João Gilberto Ponciano Pereira
Não sei se pode ser simples assim mas...
 
supondo Que um polinômio tenha grau N, o número máximo de raízes será N. E
como as funções do item 1 e item 2 tem infinitas raízes, não pode ser um
polinômio.
 
Já para o item 3, estou assumindo que um polinômio de grau N é sempre
derivável, resultando um polinômio de grau N-1, que por sua vez também é
derivável. Ora, a primeira derivada d e H(x) resulta numa função não
contínua, o que anula a hipótese.
 
 -Original Message-
From: Cláudio (Prática) [mailto:[EMAIL PROTECTED]
Sent: Monday, May 31, 2004 2:17 PM
To: [EMAIL PROTECTED]
Subject: [obm-l] Cosseno não é Polinômio



Achei estas questões interessantes:
 
Prove que as seguintes funções de R em R não são funções polinomiais:
 
1) f(x) = cos(x).
 
2) g(x) = x*sen(x)
 
3) h(x) = [x] - x + 2, onde [x] = maior inteiro menor ou igual a x. 
 
[]s,
Claudio.


=
Instruções para entrar na lista, sair da lista e usar a lista em
http://www.mat.puc-rio.br/~nicolau/olimp/obm-l.html
=


RE: [obm-l] Cosseno nao eh polinomio (2)

2004-06-01 Por tôpico João Gilberto Ponciano Pereira
Olha, sem derivadas, eu pensei em usar séries de Fourrier... e provar que o
grau do poliômio seria infinito.

-Original Message-
From: Johann Peter Gustav Lejeune Dirichlet
[mailto:[EMAIL PROTECTED]
Sent: Tuesday, June 01, 2004 1:57 PM
To: [EMAIL PROTECTED]
Subject: Re: [obm-l] Cosseno nao eh polinomio (2)


Simples: as derivadas de cos nunca sao identicamente nulas num intrvalo
aberto de IR, ao contrario das derivadas de ordem grande o bastante de um
polinomio.
 
Mas essa e uma soluçao que se usa de derivadas. Sera que nao tem algo menos
apelador?
 
Claudio Buffara [EMAIL PROTECTED] wrote:

Uma versao um pouco mais dificil:

Sejam a e b numeros reais com a  b.
Prove que F:[a,b] - R dada por F(x) = cos(x)
nao eh uma funcao polinomial.

Dessa vez o argumento de infinitas raizes nao se aplica...

[]s,
Claudio.

=
Instruções para entrar na lista, sair da lista e usar a lista em
http://www.mat.puc-rio.br/~nicolau/olimp/obm-l.html
=



TRANSIRE SVVM PECTVS MVNDOQVE POTIRI 

CONGREGATI EX TOTO ORBE MATHEMATICI OB SCRIPTA INSIGNIA TRIBVERE 

Fields Medal(John Charles Fields)
 
N.F.C. (Ne Fronti Crede)




  _  

 http://br.rd.yahoo.com//mail_br/tagline/?http://mail.yahoo.com.br/ Yahoo!
Mail - Participe da pesquisa global sobre o Yahoo! Mail. Clique
http://br.rd.yahoo.com//mail_br/tagline/?http://br.surveys.yahoo.com/global
_mail_survey_br aqui!


=
Instruções para entrar na lista, sair da lista e usar a lista em
http://www.mat.puc-rio.br/~nicolau/olimp/obm-l.html
=


RE: [obm-l] Cosseno nao eh polinomio (2)

2004-06-01 Por tôpico João Gilberto Ponciano Pereira
Só não entendi uma coisa Pq se cos(x) é Não é um polinômio, cos(k*x)
também não o será? Tá, intuitivamente isto é óbvio, mas e para provar isso?
E se k=0??? Daí cos(k*x) seria um polinômio!

-Original Message-
From: Claudio Buffara [mailto:[EMAIL PROTECTED]
Sent: Tuesday, June 01, 2004 3:28 PM
To: [EMAIL PROTECTED]
Subject: Re: [obm-l] Cosseno nao eh polinomio (2)


Que tal isso aqui?

Se cos:[a,b] - R eh uma funcao polinomial de grau n, entao a funcao:
F: [a,b] - R dada por F(x) = cos(mx) (m inteiro e fixo) 
tambem serah uma funcao polinomial de grau n em x.
Mas se n for grande o suficiente, F terah mais do que n raizes em [a,b] ==
contradicao.

[]s,
Claudio.

on 01.06.04 13:57, Johann Peter Gustav Lejeune Dirichlet at
[EMAIL PROTECTED] wrote:



Simples: as derivadas de cos nunca sao identicamente nulas num intrvalo
aberto de IR, ao contrario das derivadas de ordem grande o bastante de um
polinomio.

Mas essa e uma soluçao que se usa de derivadas. Sera que nao tem algo menos
apelador?

Claudio Buffara [EMAIL PROTECTED] wrote:


Uma versao um pouco mais dificil:

Sejam a e b numeros reais com a  b.
Prove que F:[a,b] - R dada por F(x) = cos(x)
nao eh uma funcao polinomial.

Dessa vez o argumento de infinitas raizes nao se aplica...

[]s,
Claudio.



=
Instruções para entrar na lista, sair da lista e usar a lista em
http://www.mat.puc-rio.br/~nicolau/olimp/obm-l.html
=


RE: [obm-l] Problemas em Aberto

2004-06-02 Por tôpico João Gilberto Ponciano Pereira
Acho que dá para pensar assim:

AB = lado a
BC = lado b
CD = lado c
DA = lado d

A área do quadrilátero pode ser calculada como a área do triângulo ABC +
área do triângulo ACD.

Vamos supor que conhecemos a configuração final, de área máxima, apenas para
os pontos ABC. Ou seja, dada qualquer configuração do triângulo ABC, como o
comprimento CD é fixo, fica fácil ver que a posição do ponto D para a área
máxima vai ocorrer apenas quando o triângulo ACD for retângulo em C. Daí
fica fácil, basta repetir o mesmo raciocínio utilizando como referência a
outra diagonal.

-Original Message-
From: Eric [mailto:[EMAIL PROTECTED]
Sent: Wednesday, June 02, 2004 1:21 PM
To: [EMAIL PROTECTED]
Subject: [obm-l] Problemas em Aberto


2. Tres lados consecutivos de um quadrilatero convexo sao a, b e c.
Determine o quadrilatero de area maxima .

Fiz um programa em Maple que dados os lados a,b,c
(em ordem) do quadrilatero, encontra os angulos x e y
entre os lados a,b  e b,c respectivamente que tornam
a area do quadrilatero maxima. Alem disso o programa
encontra essa area maxima e faz dois tipos diferentes
de testes para verificar o resultado. Um deles
comparando a area maxima encontrada com 1000 areas
calculadas aleatoriamente para diferentes valores de
x e y entre Pi/2 e Pi. Alem desse teste tambem uso o teste da derivada
segunda para funcoes de duas variaveis, mas
por algum motivo que nao compreendo totalmente este teste
nao funciona sempre (por exemplo, nao funciona para (a,b,c)=(1,1,1), onde
x=y=2Pi/3, nem para (a,b,c)=(1,1,6)) (pode ser algum erro
de programacao minha).

Segue abaixo alguns resultados encontrados pelo programina
(se alguem achar um erro me comunique para que eu faca as
correcoes no programa).

Fora o primeiro, os outros valores estao
aproximados (os angulos x, y estao em radianos).

(a,b,c)(x,y)Area maxima encontrada
(1,1,1)  (2Pi/3,2Pi/3) 3*(3^(1/2))/4
(1,2,1)  (1.94553; 1.94553)2.20183
(1,2,3)  (2.38820; 1.81638)4.90482
(2,3,1)  (1.81638; 2.07859)4.90482
(1,1,6)  (2.82363; 1.72977)6.08065

para (a,b,c) = (1,1,1) ou (1,1,6) o teste fornece
valores que nao passam no teste da derivada segunda
(usa uma matriz hessiana 2x2 (acho que o nome eh
esse)). Minha duvida eh justamente essa: sera que
as primeira e ultima areas encontradas sao mesmo
maximas? Elas passam no teste de comparacao com
1000 areas calculadas aleatoriamente, mas nao passam
no teste da derivada segunda...

OBS: teste da derivada segunda para funcoes de
duas variaveis:

(f_xx)(f_yy) - (f_xy)^2  0 e f_xx  0 == maximo local de f

[ ]'s

Eric.

=
Instruções para entrar na lista, sair da lista e usar a lista em
http://www.mat.puc-rio.br/~nicolau/olimp/obm-l.html
=

=
Instruções para entrar na lista, sair da lista e usar a lista em
http://www.mat.puc-rio.br/~nicolau/olimp/obm-l.html
=


RE: [obm-l] obm 2004?

2004-09-13 Por tôpico João Gilberto Ponciano Pereira
Seja f(x) = 3(x^4) - 4(x^3) e g(x) = Ax + B
traçando as duas equações em um gráfico, fica evidente que f(x) - g(x) gera
um terceiro polinômio de grau 4 com 2 pares de raízes iguais. Em outras
palavras:
3x4 - 4x^3 - Ax - B = M (x - N)^2 (x-O)^2
 
Expandindo a segunda parte e igualando aos coeficientes dos polinômios,
temos que:
em x^4: M = 3
em x^3: M(-2O - 2N) = -4
em x^2: M(O^2 + N^2 + 4ON) = 0
 
em x^1: -A= M (-2 N^2 O - 2 N O^2)
em x^0: -B = M (N^2 O^2)
 
Com as 3 primeiras equações, obtém-se os valores de M, O e N e nas duas de
baixo os valores de A e B.
 
SDS
JG

-Original Message-
From: Guilherme Pimentel [mailto:[EMAIL PROTECTED]
Sent: Monday, September 13, 2004 4:50 AM
To: [EMAIL PROTECTED]
Subject: [obm-l] obm 2004?



Determine a equação da reta que tangencia a curva de equação 
y = 3(x^4) - 4(x^3) em dois pontos distintos.
 
esta estava na obm deste ano?
 
Qq ajuda é bem vinda.
 
[]'s Guilherme



 http://www.incredimail.com/redir.asp?ad_id=322lang=22   IncrediMail - O
mundo do e-mail finalmente desenvolveu-se -
http://www.incredimail.com/redir.asp?ad_id=322lang=22 Clique aqui

attachment: IMSTP.gif

RE: [obm-l] obm 2004?

2004-09-13 Por tôpico João Gilberto Ponciano Pereira
Largando de preguiça e fazendo as contas, a equação da reta é:
 
 -8/9 x - 4/27
 
SDS
JG

-Original Message-
From: João Gilberto Ponciano Pereira 
Sent: Monday, September 13, 2004 10:58 AM
To: '[EMAIL PROTECTED]'
Subject: RE: [obm-l] obm 2004?


Seja f(x) = 3(x^4) - 4(x^3) e g(x) = Ax + B
traçando as duas equações em um gráfico, fica evidente que f(x) - g(x) gera
um terceiro polinômio de grau 4 com 2 pares de raízes iguais. Em outras
palavras:
3x4 - 4x^3 - Ax - B = M (x - N)^2 (x-O)^2
 
Expandindo a segunda parte e igualando aos coeficientes dos polinômios,
temos que:
em x^4: M = 3
em x^3: M(-2O - 2N) = -4
em x^2: M(O^2 + N^2 + 4ON) = 0
 
em x^1: -A= M (-2 N^2 O - 2 N O^2)
em x^0: -B = M (N^2 O^2)
 
Com as 3 primeiras equações, obtém-se os valores de M, O e N e nas duas de
baixo os valores de A e B.
 
SDS
JG

-Original Message-
From: Guilherme Pimentel [mailto:[EMAIL PROTECTED]
Sent: Monday, September 13, 2004 4:50 AM
To: [EMAIL PROTECTED]
Subject: [obm-l] obm 2004?



Determine a equação da reta que tangencia a curva de equação 
y = 3(x^4) - 4(x^3) em dois pontos distintos.
 
esta estava na obm deste ano?
 
Qq ajuda é bem vinda.
 
[]'s Guilherme



 http://www.incredimail.com/redir.asp?ad_id=322lang=22   IncrediMail - O
mundo do e-mail finalmente desenvolveu-se -
http://www.incredimail.com/redir.asp?ad_id=322lang=22 Clique aqui

attachment: IMSTP.gif

[obm-l] RE: [obm-l] 0 é natural? (era: Questão_4)

2004-09-14 Por tôpico João Gilberto Ponciano Pereira
Acho que o grande problema da matemática é que, às vezes, nos prendemos
muito às teorias e definições, esquecendo às vezes das utilidades práticas
ou dos motivos pelos quais estamos estudando determinado tópico.

Eu, particularmente na minha modesta opinião, sempre encarei o conjunto N
como função básica para a contagem. Por exemplo, no dia a dia, a pergunta
Quantas pedras existem nesta caixa, um leigo, sem ao menos saber, utiliza
o conjunto N como resposta. Um conjunto de variáveis discretas, palpável.

Nesta caixa existem 2 pedras
Nesta caixa existem 10 pedras
Ou então, perfeitamente possível... Nesta caixa não existe nenhuma pedra

Mesmo relacionando à história da pedrinha com carneirinhos, ou seja, uma
pedrinha para cada carneirinho, o zero ERA utilizado, mesmo que não se
entendesse na época. Uma pedrinha para cada carneirinho... Quando as
pedrinhas acabavam, quantos carneirinho existiam?

Enfim, se for para votar, concordo com o Nicolau.


-Original Message-
From: Nicolau C. Saldanha [mailto:[EMAIL PROTECTED]
Sent: Tuesday, September 14, 2004 10:00 AM
To: [EMAIL PROTECTED]
Subject: [obm-l] 0 é natural? (era: Questão_4)


On Mon, Sep 13, 2004 at 09:32:43PM -0300, Douglas Drumond wrote:
 Não quero botar lenha na fogueira, mas dois raciocínios que nos
 levam a excluir o 0 dos naturais:

Acho que o tema é legítimo, mas eternamente inconclusivo,
poderíamos falar sobre ele por tempo indeterminado.
Aliás há várias mensagens relacionadas nos arquivos.
Vou comentar os seus dois itens trocando a ordem:

 - agora um mais matemático: nos axiomas de Peano, define-se o 1 (o um
 existe). Para obter um sucessor de um númerio natural, adiciona-se 1 a
 ele. Então, se o 1 está definido, podemos adicionar 1 ao 1 p/ obter 2
 e assim por diante. Agora se os naturais começam do 0, para obter o
 sucessor de 0, adiciona-se 1. Mas o 1 não foi definido (nesse caso,
 assumimos o 0 como início e não definimos o 1), então não podemos
 adicionar 1 a ninguém.

De fato Peano é um dos maiores responsáveis pela confusão de
0 ser ou não natural: em ocasiões diferentes ele usou as duas
convenções diferentes! 

A sua forma de descrever a construção de Peano não é a usual.
Normalmente, a função sucessor é considerada mais básica do que a soma
então o seu raciocínio não se aplica. Acho um pouco artificial
supor que no início a soma já existe mas os números ainda não.

A construção dos naturais em teoria dos conjuntos, por outro lado,
começa com 0 = {} e continua com 1 = {{}}, 2 = {{},{{}}}, ...

 - vc aprende a contar a partir do 1, a história da pedrinha com
 carneirinhos começa do um (não havia a pedra zero), logo o
 modo natural de contar começa do um. Esse não é um raciocínio
 matemático, mas ajuda a memorizar que devemos iniciar a partir do 1.

Por outro lado, nada impede de chamarmos os três porquinhos de P0, P1 e P2.

O que eu acho que você deve estar dizendo é que de fato na história
o número 0 aparece muito depois de 1, 2, 3. É fato, mas também é fato
que 1 aparece depois de 2 e 3, e não antes. Mesmo em Euclides o número 1
recebe um tratamento especial que para nós parece desnecessário.

Finalmente, não se trata de memorizar nada. Estamos, espero,
discutindo uma das duas questões abaixo:

(a) Existe uma convenção razoavelmente ampla quanto a se N = {0, 1, 2,...}
ou se N = {1, 2, 3, ...}?

Acho que a resposta é claramente NÃO.

Em particular, ao redigir uma prova acho que nunca devemos dizer naturais
sem especificar qual das duas definições temos em mente. O melhor mesmo
é falar de inteiros positivos ou inteiros não negativos.

Em algumas áreas, por outro lado, os matemáticos parecem ter chegado
a um certo consenso. Em livros de teoria dos conjuntos, por exemplo,
acho que é bem universal que 0 é natural.

(b) Qual dos dois conjuntos {0, 1, 2, ...} ou {1, 2, 3, ...} merece ganhar
o nome de N, o conjunto dos naturais?

Aqui os argumentos são inconclusivos e pessoas diferentes têm opiniões
diferentes. Os dois conjuntos são claramente importantes. Se for para
votar, eu voto em N = {0, 1, 2, ...}.

[]s, N.
=
Instruções para entrar na lista, sair da lista e usar a lista em
http://www.mat.puc-rio.br/~nicolau/olimp/obm-l.html
=

=
Instruções para entrar na lista, sair da lista e usar a lista em
http://www.mat.puc-rio.br/~nicolau/olimp/obm-l.html
=


[obm-l] RE: [obm-l] Regras aritméticas

2004-09-14 Por tôpico João Gilberto Ponciano Pereira
100 * 9 = 900

100 - 11
89x Acho que não deu certo...

-Original Message-
From: Marcos Paulo [mailto:[EMAIL PROTECTED]
Sent: Tuesday, September 14, 2004 1:21 AM
To: [EMAIL PROTECTED]
Subject: Re: [obm-l] Regras aritméticas


Olá amigos da lista,
me deparei com umas regras aritméticas e
gostaria de saber o porquê delas, por exemplo:
*Multiplicação por nove: tome um número,
exemplo, 355. Pegue o trinta e cinco (centena e
dezena), adiciona um e subtrai de 355. Ou
seja,   355 - 36 = 319. Ao 319, para finalizar,
adicione um algarismo à casa das unidades para
que a soma dos algarismos dê um número múltiplo
de nove, ou seja 3195. Esse é o resultado!!

Isso é o mesmo que 3550 - 360 + 5 = 10*355 - 365 = 9*355

[]'s MP





=
Instruções para entrar na lista, sair da lista e usar a lista em
http://www.mat.puc-rio.br/~nicolau/olimp/obm-l.html
=

=
Instruções para entrar na lista, sair da lista e usar a lista em
http://www.mat.puc-rio.br/~nicolau/olimp/obm-l.html
=


[obm-l] RE: [obm-l] Como esplicar? Dúvida elementar

2004-09-15 Por tôpico João Gilberto Ponciano Pereira
Acredito que o grande problema da questão é o enunciado em si.

O fato é que se existe uma sequencia de 3 inteiros (N-2, N e N+2), cuja soma
é um determinado M também inteiros, essa sequencia é única, independente de
ser ímpar ou par. Quando falamos em 3 pares consecutivos, é como se
estivéssemos fazendo um sistema de 3 equações e 2 incógnitas.


-Original Message-
From: Qwert Smith [mailto:[EMAIL PROTECTED]
Sent: Wednesday, September 15, 2004 10:39 AM
To: [EMAIL PROTECTED]
Subject: Re: [obm-l] Como esplicar? Dúvida elementar


Eu teria escrito igualzinho ao aluno...
Na verdade acho ate ki so caberia essa preocupacao toda de saber
as reais intencoes se ele tivesse escrito 2x + 2x + 2 + 2x + 4 = 96
pq essa equacao nao eh a equacao certa para o problema proposto
e me surpreende que o professor esperasse por ela.

Se vc tivesse perguntado a metade do menor numero tudo bem.

a soma de 3 numeros impares consecutivos eh 99...

vc estaria esparando qual equacao?
2x + 1 + 2x + 1 + 2 + 2x + 1 + 4 = 99 ?

Se a e b sao numeros consecutivos de paridades iguas entao
|a-b| = 2, independente da paridade..isso o aluno entendeu
do enunciado e aplicou corretamente.


From: Daniel S. Braz [EMAIL PROTECTED]
Reply-To: [EMAIL PROTECTED]
To: [EMAIL PROTECTED]
Subject: Re: [obm-l] Como esplicar? Dúvida elementar
Date: Wed, 15 Sep 2004 00:38:32 -0300

Está correto..ele não precisava se preocupar com o fato de x ser par,
ou melhor escrito como 2x..já que segundo o próprio enuncionado os 3
números são par..
se x é par (segundo o enunciado), x + 2 e x + 4 também serão..eu daria
10 na prova!!! ;)

agora...talvez seja bom vc verificar com ele qual era a real
intenção..se ele realmente pensou nisso ou saiu sem querer..só para
evitar futuros problemas..

[]s
daniel


On Tue, 14 Sep 2004 23:58:47 -0300, Fernando Aires
[EMAIL PROTECTED] wrote:
  Gustavo,
 
 Opinião pedagógica: ele encontrou uma solução correta.
 Pense assim: se ele dissesse O menor deles é o 30, porque
  30+32+34=96, não estaria correto? Ele precisava achar o menor deles,
  e achou, e provou por que achou...
 Ele chegou num x=30, que é par... E se ele pensou no fato de, como
  a soma é par, e todos os números têm a mesma paridade, então cada um
  deles é par - e, portanto, não é preciso se preocupar com x ser par?
  Eu daria nota máxima (já tive alguns casos semelhantes em classe),
  porque ele provou razoavelmente o que escreveu (é diferente de
  escrever 30 simplesmente - eu já aviso no começo da prova que, se
  fizer isso, eu nem considero a questão). E eu comentaria por escrito o
  fato de que seria legal você ter se preocupado em o x ser par mais
  explicitamente, dizendo 2x + 2x+2 + 2x+4 = 96 (num linguajar de
  menino de sétima série, claro...).
 
  Beijos,
 
  --
  --
  Fernando Aires
  [EMAIL PROTECTED]
  Em tudo Amar e Servir
  --
 
 
 
  - Original Message -
  From: Gustavo [EMAIL PROTECTED]
  Date: Wed, 15 Sep 2004 00:01:48 -0300
  Subject: [obm-l] Como esplicar? Dúvida elementar
  To: Olímpiada [EMAIL PROTECTED]
 
  Passei o seguinte problema para um garoto da sétima série : a soma de
  3 números pares consecutivos é igual a 96, qual o menor deles?
  esperava eu a seguinte equação: 2x +2x +2 +2x +4 = 96,  2x=30 , no
  entanto encontrei;x+x+2+x+4 =96  x=30, Devo considerar a equação
  dele correta ? Usando que argumentos ? em uma prova devo considerar
  certo ou errado ? Alguém pode opinar??Obrigado.
 
  
=
  Instruções para entrar na lista, sair da lista e usar a lista em
  http://www.mat.puc-rio.br/~nicolau/olimp/obm-l.html
  
=
 



--
Uma das coisas notáveis acerca do comportamento do Universo é que ele
parece fundamentar-se na Matemática num grau totalmente
extraordinário. Quanto mais profundamente entramos nas leis da
Natureza, mais parece que o mundo físico quase se evapora e ficamos
com a Matemática. Quanto mais profundamente entendemos a Natureza,
mais somos conduzidos para dentro desse mundo da Matemática e de
conceitos matemáticos. (Roger Penrose)

=
Instruções para entrar na lista, sair da lista e usar a lista em
http://www.mat.puc-rio.br/~nicolau/olimp/obm-l.html
=

_
On the road to retirement? Check out MSN Life Events for advice on how to 
get there! http://lifeevents.msn.com/category.aspx?cid=Retirement

=
Instruções para entrar na lista, sair da lista e usar a lista em
http://www.mat.puc-rio.br/~nicolau/olimp/obm-l.html
=

=
Instruções para entrar na lista, 

[obm-l] Cubo mágico

2004-09-15 Por tôpico João Gilberto Ponciano Pereira
1- Imaginem 125 cubos iguais, numerados sequencialmente de 1 a 125. Prove
que é possível rearranjá-los, formando um cubo 5x5x5, de tal forma que
qualquer fatia  5x5x1 do cubo apresente a soma dos números dos cubinhos
constante.

2- Para quais valores de N é possível montar um cubo NxNxN com as mesmas
propriedades do item anterior?

=
Instruções para entrar na lista, sair da lista e usar a lista em
http://www.mat.puc-rio.br/~nicolau/olimp/obm-l.html
=


RE: [obm-l] inteiros

2004-09-21 Por tôpico João Gilberto Ponciano Pereira
Hum... Vamos de um jeito mais bonito então

Chamando Mod(k^5,10) = M (M é o resto da divisão de k^5 por M)

Quando k = 0, M=0

Sabemos também que:
(K+1)^5 = K^5 + 5*k^4 + 10*K^3 + 10*K^2 + 5*K + 1
(K+1)^5 = K^5 + 5*k*(k^3+1) + 10*K^2(K+1) + 1

Observem que o termo 5*k*(k^3+1) será sempre múltiplo de 10 para k inteiro.
(Se k é impar, k^3+1 é par)

Tirando o módulo da divisão por 10 de tudo isso, temos:
mod((k+1)^5) = mod(k^5) + 1
 e como mod(0) = 0

-Original Message-
From: Fernando Aires [mailto:[EMAIL PROTECTED]
Sent: Tuesday, September 21, 2004 5:59 PM
To: [EMAIL PROTECTED]
Subject: Re: [obm-l] inteiros


Hermann,

   Eu tenho uma idéia:

   Pelo método da multiplicação, sabemos que a unidade resultante
depende apenas de uma operação, que é a multiplicação dos algarismos
das unidades dos fatores. Desta forma, podemos provar diretamente para
cada um dos possíveis algarimos das unidades ([0;9]).

   Para o número x=ABC...N0:
   ABC...N0 ^2 = A'B'C'...N'0 (x*x=x^2)
   A'B'C'...N'0 * ABC...N0 = A''B''C''...N''0 (x^2 * x = x^3)
   A''B''C''...N''0 * A'B'C'...N'0 = A'''B'''C'''...N'''0 (x^3 * x^2 = x^5)

   Para o número x=ABC...N1:
   ABC...N01^2 = A'B'C'...N'1 (x*x=x^2)
   A'B'C'...N'1 * ABC...N1 = A''B''C''...N''1 (x^2 * x = x^3)
   A''B''C''...N''1 * A'B'C'...N'1 = A'''B'''C'''...N'''1 (x^3 * x^2 = x^5)

   Para o número x=ABC...N2:
   ABC...N2 ^2 = A'B'C'...N'4 (x*x=x^2)
   A'B'C'...N'4 * ABC...N2 = A''B''C''...N''8 (x^2 * x = x^3)
   A''B''C''...N''4 * A'B'C'...N'8 = A'''B'''C'''...N'''2 (x^3 * x^2 = x^5)

   Para o número x=ABC...N3:
   ABC...N3 ^2 = A'B'C'...N'9 (x*x=x^2)
   A'B'C'...N'9 * ABC...N3 = A''B''C''...N''7 (x^2 * x = x^3)
   A''B''C''...N''7 * A'B'C'...N'9 = A'''B'''C'''...N'''3 (x^3 * x^2 = x^5)

   Para o número x=ABC...N4:
   ABC...N4 ^2 = A'B'C'...N'6 (x*x=x^2)
   A'B'C'...N'6 * ABC...N4 = A''B''C''...N''4 (x^2 * x = x^3)
   A''B''C''...N''4 * A'B'C'...N'6 = A'''B'''C'''...N'''4 (x^3 * x^2 = x^5)

   Para o número x=ABC...N5:
   ABC...N5 ^2 = A'B'C'...N'5 (x*x=x^2)
   A'B'C'...N'5 * ABC...N5 = A''B''C''...N''5 (x^2 * x = x^3)
   A''B''C''...N''5 * A'B'C'...N'5 = A'''B'''C'''...N'''5 (x^3 * x^2 = x^5)

   Para o número x=ABC...N6:
   ABC...N6 ^2 = A'B'C'...N'6 (x*x=x^2)
   A'B'C'...N'6 * ABC...N6 = A''B''C''...N''6 (x^2 * x = x^3)
   A''B''C''...N''6 * A'B'C'...N'6 = A'''B'''C'''...N'''6 (x^3 * x^2 = x^5)

   Para o número x=ABC...N7:
   ABC...N7 ^2 = A'B'C'...N'9 (x*x=x^2)
   A'B'C'...N'9 * ABC...N7 = A''B''C''...N''3 (x^2 * x = x^3)
   A''B''C''...N''3 * A'B'C'...N'9 = A'''B'''C'''...N'''7 (x^3 * x^2 = x^5)

   Para o número x=ABC...N8:
   ABC...N8 ^2 = A'B'C'...N'4 (x*x=x^2)
   A'B'C'...N'4 * ABC...N8 = A''B''C''...N''2 (x^2 * x = x^3)
   A''B''C''...N''2 * A'B'C'...N'4 = A'''B'''C'''...N'''8 (x^3 * x^2 = x^5)

   Para o número x=ABC...N9:
   ABC...N9 ^2 = A'B'C'...N'1 (x*x=x^2)
   A'B'C'...N'1 * ABC...N9 = A''B''C''...N''9 (x^2 * x = x^3)
   A''B''C''...N''9 * A'B'C'...N'1 = A'''B'''C'''...N'''9 (x^3 * x^2 = x^5)

   (C.Q.D.)

   Apesar de não muito elegante, é uma demonstração válida.
   (Mas estou pensando na prova que K^5-K é múltiplo de 10)

Beijos,

-- 
--
Fernando Aires
[EMAIL PROTECTED]
Em tudo Amar e Servir
--

On Tue, 21 Sep 2004 17:03:05 -0300, Tio Cabri st
[EMAIL PROTECTED] wrote:
 Por favor...
 Como demonstro o seguinte:
 
 Se K é um número Natural então K^5 possui o mesmo algarismo das unidades.
 
 TEntei fazer por indução empaquei.
 Tentei demonstrar que k^5-K é múltiplo de dez empaquei novamente
 
 espero que alguém da lista saiba
 Obrigado,
 Hermann

=
Instruções para entrar na lista, sair da lista e usar a lista em
http://www.mat.puc-rio.br/~nicolau/olimp/obm-l.html
=

=
Instruções para entrar na lista, sair da lista e usar a lista em
http://www.mat.puc-rio.br/~nicolau/olimp/obm-l.html
=


RE: [obm-l] amigos do PONCE

2004-09-22 Por tôpico João Gilberto Ponciano Pereira
De um jeito chato:
720 = 2^4 * 3^2 * 5

os divisores de 720 serão todas as combinações de 2^n * 3^m * 5^o, com n,m,o
=0 e menor ou igual a, respectivamente, 4,2 e 1.
Bem, vamos chamar a soma das combinações de 5 de S1 = 5^1 + 5^0 = 6
seja S2 a soma das combinações de 3 e 5  -  S2 = S1*(3^2+3^1+3^0) = 6*13 =
78
e finalmente, pelo mesmo raciocínio, S3 = S2 *(2^4 + 2^3+2^2+2^1+2^0) = 78 *
31 = 2418

sds
JG
-Original Message-
From: Marcelo Majewski [mailto:[EMAIL PROTECTED]
Sent: Wednesday, September 22, 2004 2:15 PM
To: obm-l
Subject: [obm-l] amigos do PONCE


estou com problema e nao sei resolver fazendo alguma 
relacao...

quanto vale a soma de todos os divisores de 720?
 
__
Acabe com aquelas janelinhas que pulam na sua tela.
AntiPop-up UOL - É grátis!
http://antipopup.uol.com.br/



=
Instruções para entrar na lista, sair da lista e usar a lista em
http://www.mat.puc-rio.br/~nicolau/olimp/obm-l.html
=

=
Instruções para entrar na lista, sair da lista e usar a lista em
http://www.mat.puc-rio.br/~nicolau/olimp/obm-l.html
=


RE: [obm-l] OBM - 03

2004-09-22 Por tôpico João Gilberto Ponciano Pereira
Não sei se esta foi a idéia do Cláudio, mas vamos tentar...
f(x) = x2 + 5x + 23, para x inteiro, o menos valor de f(x) = 17 quando x=-2
 
seja d(x) = f(x+1) - f(x) = (x2 + 2x+1) - x2 + (5x + 5) - 5x +23 - 23 = 2x +
6
d(-2) = 2
d(-1) = 4
d(0) = 6
...
 
Logo, os possíveis valores de f(x) serão da forma 17 + (2+4+6++2*m) = 17
+ 2*(1+2+3+...+m) = 17 + m*(m+1)
daí, basta analisar os módulos ara cada primo
 
tirando o módulo, temos:
Termo1Termo 2
mod(17)  + mod(m) * mod(m+1) 
 
os valores possíveis para o termo 2 são:
0; 2; 6; 12; 20; 30 (0*1, 1*2, 2*3, ...) 
 
para primo 2, termo 1 = 1 e termo 2 = 0
para primo 3, termo 1 = 2 e termo 2 = 0 ou 2
para primo 5, termo 1 = 2 e termo 2 = 0, 2, 1 
para primo 7, termo 1 = 3 e termo 2 = 0, 2, 6, 5
para primo 11, termo 1 = 6 e termo 2 = 0, 2, 6, 1, 9, 8
para primo 13, termo 1 = 4 e termo 2 = 0, 2, 6, 12, 7, 4, 3
Logo, 17 é o menor primo..
 
sds
jg
 
-Original Message-
From: claudio.buffara [mailto:[EMAIL PROTECTED]
Sent: Wednesday, September 22, 2004 2:44 PM
To: obm-l
Subject: Re:[obm-l] OBM - 03



 
De:  [EMAIL PROTECTED]  
Para:[EMAIL PROTECTED]  
Cópia:  
Data:Wed, 22 Sep 2004 15:08:31 +
Assunto: [obm-l] OBM - 03   

 Queria q vcs me ajudassem nessa aqui ...
 
 Determine o menor número primo positivo que divide x^2 + 5x + 23 para
algum 
 inteiro x.
 
Dica: Inicialmente faça algumas explorações numéricas com valores inteiros
de x em torno do ponto de mínimo de f(x) = x^2 + 5x + 23 a fim de obter uma
conjectura.
Para provar esta conjectura, lembre-se de que se, para algum inteiro x, f(x)
é divisível por n, então se você tomar n valores inteiros consecutivos de x,
algum dos f(x) correspondentes será divisível por n (por que?).
 
[]s,
Claudio.
 


=
Instruções para entrar na lista, sair da lista e usar a lista em
http://www.mat.puc-rio.br/~nicolau/olimp/obm-l.html
=


RE: [obm-l] Mais um problema legal

2004-10-13 Por tôpico João Gilberto Ponciano Pereira
Imaginei assim:

Sejam as 3 parcelas X, Y e Z tais que X = Y= Z. Logo, X=667.

1- supondo X fixo ímpar, temos que Y pode variar de X (inclusive) a (2002 -
1 - x)/2. Em outras palavras, quando X=1 temos (2002 - 1 - 1)/2 = 1000
valores possíveis para Y válidos, logo, 1000 possíveis variações. Colocando
numa tabela para facilitar as contas, temos:

  X  - Variações
  1  -  1000
  3  -   997
  5  -   994
..
667  - 1

2- analogamente, supondo um x fixo par, temos que Y pode variar de X
(inclusive) a (2002 - x)/2. Neste caso, a tabela ficaria com a seguinte
cara:

  X  - Variações
  2  -   999
  4  -   996
  6  -   993
..
666  - 3

Fazendo as contas necessárias, chego num número um pouco diferente: 
334000


SDS
JG

-Original Message-
From: [EMAIL PROTECTED] [mailto:[EMAIL PROTECTED]
Sent: Wednesday, October 13, 2004 3:22 PM
To: [EMAIL PROTECTED]
Subject: RES: [obm-l] Mais um problema legal


Na realidade a quantidade de somas que têm o zero em uma das parcelas é
1002, o que nos dá
[2004!/(2002! * 2! * 3!)] - 1002.

Acho que agora está correto.

-Mensagem original-
De: [EMAIL PROTECTED] [mailto:[EMAIL PROTECTED]
Enviada em: terça-feira, 12 de outubro de 2004 21:21
Para: [EMAIL PROTECTED]
Assunto: RES: [obm-l] Mais um problema legal


Penso que uma boa seqüência de resolução desse problema seja [2004!/(2002! *
2! * 3!)] - 2003, pois para obtermos 2002 como a soma de três inteiros
positivos, podemos ter

|...||| + |||...|| + |||...|| = 2002
 500600 902

como também

|||...|| + ||...|| + |||...||| = 2002
   200 1000 802

Portanto teríamos todas as permutações possíveis dos 2004 símbolos
(tracinhos e sinais de +), dividido pelas permutações dos tracinhos e também
dos sinais. A divisão pela permutação de 3 é porque cada soma, em função da
ordem não importar, tem 3! repetições. A diminuição de 2003 possibilidades é
nececssária para que se retirem as soluções que têm o zero em uma das
parcelas.

Sou um novo integrante do grupo e pela primeira vez tento enviar alguma
resposta. Espero que consiga. Caso haja equívocos na minha solução, espero
respostas.

Um abraço a todos.

Agamenon.


-Mensagem original-
De: [EMAIL PROTECTED] [mailto:[EMAIL PROTECTED]
nome de benedito
Enviada em: terça-feira, 12 de outubro de 2004 18:08
Para: [EMAIL PROTECTED]
Assunto: [obm-l] Mais um problema legal





 Segue mais um problema interessante (Agora com o problema. Desculpem a
falha).
 Benedito Freire

PROBLEMA

Sem levar em consideração a ordem, de quantas maneiras podemos expressar
2002  como soma de  3  inteiros positivos?

(Atenção: 1000 + 1000 + 3 = 2002   e  1000 + 2 + 1000 = 2002  não são
consideradas maneiras distintas de expressar  2002  como soma de inteiros
positivos)




--
Esta mensagem foi verificada pelo sistema de anti-virus e
 acredita-se estar livre de perigo.

=
Instruções para entrar na lista, sair da lista e usar a lista em
http://www.mat.puc-rio.br/~nicolau/olimp/obm-l.html
=


=
Instruções para entrar na lista, sair da lista e usar a lista em
http://www.mat.puc-rio.br/~nicolau/olimp/obm-l.html
=

=
Instruções para entrar na lista, sair da lista e usar a lista em
http://www.mat.puc-rio.br/~nicolau/olimp/obm-l.html
=


RE: [obm-l] Quadrilatero Incritivel

2004-10-28 Por tôpico João Gilberto Ponciano Pereira
Bom, se eu não me engano, o problema original era traçar o quadrilátero de
maior área dados o comprimento de 3 segmentos. 

De qualquer forma, na nova versão, acho que se pensarmos que dados os lados,
o quadrilátero de maior área é o inscritivel (tem que provar isso!!!),
talvez ajude na construção do mesmo...

-Original Message-
From: Claudio Buffara [mailto:[EMAIL PROTECTED]
Sent: Thursday, October 28, 2004 5:27 PM
To: [EMAIL PROTECTED]
Subject: Re: [obm-l] Quadrilatero Incritivel


on 28.10.04 17:01, Ricardo Bittencourt at [EMAIL PROTECTED] wrote:

 Qwert Smith wrote:
 Construir um quadrilatero inscritivel ABCD dados AB e os
 comprimentos de BC,
 CD e DA.
 Veja o link:
 http://mathworld.wolfram.com/CyclicQuadrilateral.html
 Ele te da a formula das diagonais do quadrilatero incritivel em funcao
 dos lados.
 
 Mas nesse caso você precisa de uma régua graduada,
 pra poder marcar com precisão o valor da diagonal. O problema
 fica melhor se a régua não tiver marcação alguma.
 
Concordo com a objecao: construcao com regua marcada nao vale.

No entanto, eu aceitaria uma solucao em que o Qwert demonstrasse as formulas
para os comprimentos das diagonais em funcao dos comprimentos dos lados e
mostrasse como construir segmentos iguais as diagonais com uma regua nao
marcada e um compasso.

Naturalmente, como a oferta do livro foi feita pelo Wagner, a ultima palavra
deve ser dele.

[]s,
Claudio.



=
Instruções para entrar na lista, sair da lista e usar a lista em
http://www.mat.puc-rio.br/~nicolau/olimp/obm-l.html
=

=
Instruções para entrar na lista, sair da lista e usar a lista em
http://www.mat.puc-rio.br/~nicolau/olimp/obm-l.html
=


[obm-l] RE: [obm-l] OUTRO ENUNCIADO DÚBIO!

2004-11-04 Por tôpico João Gilberto Ponciano Pereira
Este me faz lembrar o problema do carnaval Que tem solução.

Dois homens encontram duas mulheres no carnaval e todos decidem ir para a
cama. Entretanto, só haviam 2 camisinhas disponíveis. Como deve ser feito
para que cada homem tenha relações com as duas mulheres, sem correr o risco
de nenhuma doença contagiosa?? ou seja, H1 x M1, H1 x M2, H2 x M1, H2 x M2.

-Original Message-
From: [EMAIL PROTECTED] [mailto:[EMAIL PROTECTED]
Sent: Wednesday, November 03, 2004 9:09 PM
To: [EMAIL PROTECTED]
Subject: [obm-l] OUTRO ENUNCIADO DÚBIO!


Oi, Pessoal! Acho que já vi coisa parecida aqui na lista, ou pelo menos mais
compacto!

Um paciente precisava ser operado do estômago por três médicos diferentes.
Havia
apenas um par de luvas clínicas. Assolava o país uma epidemia terrível,
transmitida a quem encostasse em outra pessoa ou mesmo em qualquer objeto
tocado por terceiros. Felizmente os médicos não trabalhariam
simultaneamente,
mas em seqüencia, de modo que cada um poderia cumprir sua parte, com luvas
clínicas, sem encostar nos outros ou em objetos contaminados. Como as luvas
deveriam ser usadas?

A propósito, aqui jaz o filho, a mãe, a filha, o pai, a irmã, o irmão, a
espôsa
e também o espôso. Qual o número mínimo de pessoas sepultadas?


Abraços!



__
WebMail UNIFOR - http://www.unifor.br.
=
Instruções para entrar na lista, sair da lista e usar a lista em
http://www.mat.puc-rio.br/~nicolau/olimp/obm-l.html
=

=
Instruções para entrar na lista, sair da lista e usar a lista em
http://www.mat.puc-rio.br/~nicolau/olimp/obm-l.html
=


[obm-l] RE: [obm-l] movendo peças em linha

2004-11-10 Por tôpico João Gilberto Ponciano Pereira
Claudio

Vamos ver se entendi direito. Vamos supor N peças de cada cor. Peças da
mesma cor não se ultrapassam, correto? logo, supondo peças B1, B2, B2, BN
irão sempre ter a mesma ordem. Logo, partindo da configuração inicial até a
final, serão necessários N+1 movimentos com cada peça para que ela saia da
posição inicial e chegue na final. O total de movimentos seria (N+1)*2N.

Entretanto, podemos chamar de movimento do pulo quando uma pedra se
movimenta por cima da outra. Este movimento vale como 2 movimentos normais.
Como uma peça tem que ultrapassar N outras peças, teremos um total de N^2
pulos.

Logo, Não existe um caminho certo. Pelo que entendi, qualquer sequencia
válida é ótima, com um número de movimentos igual a N^2 + 2N. rs... para 1,
minha lógica vale... agora para o resto, tem que testar.

SDS
JG


-Original Message-
From: Claudio Buffara [mailto:[EMAIL PROTECTED]
Sent: Tuesday, November 09, 2004 10:31 PM
To: Lista OBM
Subject: [obm-l] movendo peças em linha


Um tabuleiro 1 x (2n+1) contem n peças brancas ocupando as casas 1, 2, ...,
n  e  n peças pretas ocupando as casas n+2, n+3, ..., 2n+1. A casa n+1 estah
inicialmente vazia.

O objetivo eh colocar as n peças pretas nas casas 1, 2, ..., n e as n peças
brancas nas casas n+2, n+3, ..., 2n+1.

Os movimentos permitidos sao os seguintes:
1) Para as peças brancas:
1a) Deslocamento da casa k para a casa k+1, se esta estiver vazia;
1b) Deslocamento da casa k para a casa k+2, se esta estiver vazia e a casa
k+1 contiver uma peça preta.

2) Para as peças pretas:
2a) Deslocamento da casa k para a casa k-1, se esta estiver vazia;
2b) Deslocamento da casa k para a casa k-2, se esta estiver vazia e a casa
k-1 contiver uma peça branca.

Supondo que duas peças duma mesma cor sao indistinguiveis, qual o menor
numero de movimentos necessarios para que o objetivo pode ser atingido?

Por exemplo, para n = 1, a resposta eh 3:
 1  2  3
[B][ ][P]
[B][P][ ]
[ ][P][B]
[P][ ][B]

[]s,
Claudio.


=
Instruções para entrar na lista, sair da lista e usar a lista em
http://www.mat.puc-rio.br/~nicolau/olimp/obm-l.html
=

=
Instruções para entrar na lista, sair da lista e usar a lista em
http://www.mat.puc-rio.br/~nicolau/olimp/obm-l.html
=


RE: [obm-l] Perguntas simples para respostas convincentes

2004-12-09 Por tôpico João Gilberto Ponciano Pereira
Ai, gente... apesar de achar meio off-topic, o que acontece é o seguinte:
Imaginem que circule em todo Brasil um total de R$3000, aproximadamente
US$1000. O governo decide injetar mais R$500 só para as pessoas carentes,
aumentando o poder aquisitivo delas, rodando mais dinheiro, fazendo os
preços subirem consequentemente... Ou seja, uma coca, que custava antes
R$1,00, só com este dinheiro a mais circulando, passa a custar R$1,50. Sim,
inflação... no final das contas, os R$3500 circulando pelo brasil acabam
valendo os mesmo US$1000 para o resto do mundo...

-Original Message-
From: Bernardo Freitas Paulo da Costa [mailto:[EMAIL PROTECTED]
Sent: Thursday, December 09, 2004 6:24 AM
To: [EMAIL PROTECTED]
Subject: Re: [obm-l] Perguntas simples para respostas convincentes


Oi,

Acho que o lastro-ouro já acabou há tempos. Talvez com a excessão da
Alemanha e outros poucos países, o que existe é que cada país possui
uma cesta de moedas, como por exemplo dólar, yen, libra, etc... que
definem quanto vale a moeda local.

Abraços,
-- 
Bernardo Freitas Paulo da Costa

On Thu,  9 Dec 2004 01:07:02 -0200, Osvaldo Mello Sponquiado
[EMAIL PROTECTED] wrote:
 
  
  1- Pq o Banco Central nao distribui dinheiro pro povo 
  e acaba com a miseria, ja que ele fabrica moeda 
  
   
 Até parece ki é tão facil assim... 
 Já ouviu falar o lastro-ouro ? 
 Cada centavo emitido na nação tem que ser guardado na forma de ouro, sem
 nenhuma utilização. 
   
   
   
   
   
 Atenciosamente, 
  
 Osvaldo Mello Sponquiado 
 Engenharia Elétrica, 2ºano 
 UNESP - Ilha Solteira 


=
Instruções para entrar na lista, sair da lista e usar a lista em
http://www.mat.puc-rio.br/~nicolau/olimp/obm-l.html
=

=
Instruções para entrar na lista, sair da lista e usar a lista em
http://www.mat.puc-rio.br/~nicolau/olimp/obm-l.html
=


RE: [obm-l] ENUNCIADO ORIGINAL!

2005-01-20 Por tôpico João Gilberto Ponciano Pereira
Hum... eu diria que é apenas um par, dois sapatos.
3 pares podem fazer a configuração AB, BC, CA.

-Original Message-
From: Rogerio Ponce [mailto:[EMAIL PROTECTED]
Sent: Thursday, January 20, 2005 2:28 PM
To: obm-l@mat.puc-rio.br
Subject: RE: [obm-l] ENUNCIADO ORIGINAL!


Olá Jorge,
de estalo eu diria que são 3 pares: 2 iguais e 1 diferente.
Seis sapatos, portanto.
Abraços,
Rogério.

From: jorgeluis
Date: Wed, 19 Jan 2005 21:25:00 -0300

Num hotel, os sapatos a serem engraxados são colocados do lado de fora da 
porta.
A camareira que realiza este trabalho tem que responder à pergunta sobre 
quantos
sapatos há a limpar. Então diz: Se reunirmos os sapatos em pares, combinam

ou
um só par ou todos os pares. Quantos são os sapatos?


_
MSN Hotmail, o maior webmail do Brasil.  http://www.hotmail.com

=
Instruções para entrar na lista, sair da lista e usar a lista em
http://www.mat.puc-rio.br/~nicolau/olimp/obm-l.html
=

=
Instruções para entrar na lista, sair da lista e usar a lista em
http://www.mat.puc-rio.br/~nicolau/olimp/obm-l.html
=


RE: [obm-l] Por 7

2005-04-14 Por tôpico João Gilberto Ponciano Pereira
Opa... peraí... Pelo que entendi, a regra valeria apenas para um dígito.

Veja o caso de 59768758231 (que é divisível por 7)

5976875 - 2*8231 = 5960413
596 - 2*413 = -230 (que não é divisível por 7!!!)

Isso pq essa regra funciona pois 2*10 mod 7 = -1.

Para funcionar para números grandes, podemos usar que 1000 mod 7 = -1.
Assim, usaríamos 3 dígitos, agilizando as contas, e não seria nem necessário
multiplicar por 2.

o mesmo exemplo, teríamos:
59768758 - 231 = 59768527
59768- 527 = 59241
59   - 241 = -182, que é divisível.

SDS
JG


-Original Message-
From: Johann Peter Gustav Lejeune Dirichlet
[mailto:[EMAIL PROTECTED]
Sent: Monday, April 11, 2005 3:48 PM
To: obm-l@mat.puc-rio.br
Subject: RE: [obm-l] Por 7


--Que historia e essa de so e bom para numeros
pequenos? Para comeco de historia, pode-se pegar
qualquer bloco de digitos em vez de apenas um por vez.
Ou seja, podemos fazer algo como:

5976875(8234)
- 16468
-
5960407

  596(0407)
-0407
-0407
-218

E isto nao e multiplo de 7.
(So para desencargo de consciencia, conferi na BC...)

E apenas para terminar, normalmente ninguem te
perguntaria se o numero abaixo e ou nao multiplo de 7:

597687582345976875823459768758234597687582 \
345976597687582345976875823459768758234597 \
687582345976597687582345976875823459768758 \
234597687582345976597687582345976875823459 \
768758234597687582345976875823459768758234 \
598947895789456844566496313554564654456613 \
324165456489789754123164641304104817105130 \
152895531714012404504576875823459768758234 ^ 2

/*Os \ sao apenas quebras de linha para maior
legibilidade*/
/*Ou para menor ilegibilidade, entenda como
quiser...*/

Neste caso especificamente, nenhum criterio e melhor
que o outro (ou estou muito enganado, o que
ultimamente e um fato-comum)... 

--- Rafael Alfinito Ferreira [EMAIL PROTECTED]
wrote:
 este processo só é bom para números pequenos, por
 exemplo:
 me diga se 59768758234 é divisível por 7 , aí é
 melhor usar a técnica das 
 classes.
 
 VALEU! UM ABRAÇO!
 RAFAEL FERREIRA
 





Yahoo! Acesso Grátis - Internet rápida e grátis. 
Instale o discador agora! http://br.acesso.yahoo.com/
=
Instruções para entrar na lista, sair da lista e usar a lista em
http://www.mat.puc-rio.br/~nicolau/olimp/obm-l.html
=

=
Instruções para entrar na lista, sair da lista e usar a lista em
http://www.mat.puc-rio.br/~nicolau/olimp/obm-l.html
=


[obm-l] RE: [obm-l] Questão da OBM universitá ria

2005-06-29 Por tôpico João Gilberto Ponciano Pereira
Bom... Pelo que entendi, o ponto A terá sempre coordenadas (a/2,0),
justamente pela propriedade de ser o ponto médio do segmento BP. Logo,
levando em consideração a derivada da reta, podemos dizer que (a/2)*f'(a) =
f(a).

Daí fica mais fácil... Acho que nem precisa entrar no mérito de equações
diferenciais para ver que f(a)é quadrática, no caso, levando em conta os
parâmetros do item A, f(x)=X^2/2.

-Original Message-
From: Daniel Regufe [mailto:[EMAIL PROTECTED]
Sent: Wednesday, June 29, 2005 9:45 AM
To: obm-l@mat.puc-rio.br
Subject: [obm-l] Questão da OBM universitária


queria ver uma solução dessa questão ...

A função derivável : f: R - R tem as seguintes propriedades:
a) f(0) = 0 e f(2) = 2
b) Para todo a pertencente a R\{0}, a reta tangente ao gráfico de f no 
ponto P = (a,f(a)) corta o eixo x em um ponto A e o eixo y em um ponto B de 
tal forma que A é o ponto médio do
segmento BP.
Calcule f(3).

Alguem pode me ajudar ?
[]´s
Daniel Regufe

_
MSN Messenger: converse online com seus amigos .  
http://messenger.msn.com.br

=
Instruções para entrar na lista, sair da lista e usar a lista em
http://www.mat.puc-rio.br/~nicolau/olimp/obm-l.html
=

=
Instruções para entrar na lista, sair da lista e usar a lista em
http://www.mat.puc-rio.br/~nicolau/olimp/obm-l.html
=


RE: [obm-l] numeros primos

2005-12-13 Por tôpico João Gilberto Ponciano Pereira
2^0

-Original Message-
From: [EMAIL PROTECTED] [mailto:[EMAIL PROTECTED]
Behalf Of Murilo RFL
Sent: Tuesday, December 13, 2005 1:35 PM
To: obm-l@mat.puc-rio.br
Subject: Re: [obm-l] numeros primos


Sabemos q o menor numero q pode ser representado por 3^a é 3 e por 2^b é 2
Logo 3^a sempre será impar e 2^b sempre par
como um impar - um par eh sempre impar, 2 nao pode ser representado. Sendo o

menor primo.

Bom.. talvez fossem os numeros inteiros nao negativos... mas esta ai uma 
solução

- Original Message - 
From: Rodrigo Augusto [EMAIL PROTECTED]
To: obm-l@mat.puc-rio.br
Sent: Tuesday, December 13, 2005 1:31 PM
Subject: [obm-l] numeros primos


 preciso de ajuda com essa questão:

 Qual o menor número primo P que NAO pode ser representado na forma 3^a - 
 2^b (em módulo) ? onde a e b são inteiros positivos.

 por favor, apresentem a resolucao!

 valeu

 _
 MSN Messenger: converse online com seus amigos . 
 http://messenger.msn.com.br

 =
 Instruções para entrar na lista, sair da lista e usar a lista em
 http://www.mat.puc-rio.br/~nicolau/olimp/obm-l.html
 =
 

=
Instruções para entrar na lista, sair da lista e usar a lista em
http://www.mat.puc-rio.br/~nicolau/olimp/obm-l.html
=

=
Instruções para entrar na lista, sair da lista e usar a lista em
http://www.mat.puc-rio.br/~nicolau/olimp/obm-l.html
=


RE: [obm-l] Problema

2006-01-02 Por tôpico João Gilberto Ponciano Pereira
Acho que o interessante do problema é chegar nesses números (92 e 47). Até o 
momento, a única coisa que consegui concluir é que a soma dos números não pode 
ser par, ou seja, um dos números é par e o outro é ímpar.
 
 
 -Original Message-
From: [EMAIL PROTECTED] [mailto:[EMAIL PROTECTED] Behalf Of Eduardo Wilner
Sent: Monday, January 02, 2006 12:51 PM
To: obm-l@mat.puc-rio.br
Subject: Re: [obm-l] Problema





 Prezado Garcia

  Achei o problema interessante mas, não conhecendo as velhas versões que vc. 
menciona,  não sei se é este o espírito da questão. Por favor corrija.

  O primeiro matemático  recebe o produto como sendo 4324 que pode ser fatorado 
como 2*2*23*47, sendo sua dúvida como agrupar em dois fatores. 
  Assim ele declara que o produto é insuficiente para se conhecer os dois 
fatores.
  O segundo recebe a soma como 139 sabendo então que as paridades dos dois 
números não são iguais, logo o produto seria par, e o primeiro não saberia se é 
um par vezes um impar ou um par vezes um par, e declara que já sabia  que o 
produto seria insuficiente.
  O primeiro então sabe que a soma é impar e que os números são de pardiade 
diferente: 2*2*23=92 e 47  . Declara : Então conheço os números
  O segundo (que não é bobo) faz o mesmo raciocínio que nós estamos fazendo e 
declara: Nesse caso, eu também.
  Seria isso, Garcia?

  []s

Wilner 

 
[EMAIL PROTECTED] escreveu: 


Me lembrei de outro velho problema que me passaram com dados novos:

Um gênio matemático recebe, num papel, a soma de dois números inteiros entre 2 e
100. Um outro gênio recebe o produto dos mesmos dois números. Os dois iniciam o
diálogo:

- Este produto não é o suficiente para achar os dois números.
- Eu sabia.
- Então, eu conheço estes números.
- Nesse caso, eu também.
- Quais são os dois números?





Citando Adriano Torres :

 Olá, sou novo aqui na lista, e gostaria de propor um problema para que me 
 ensinassem a solução.
 Estou enviando a figura do triângulo para que possa ser visto.
 É um triangulo isóceles, com AB = AC, ângulo bÂc = 20°, cBt = 30° e bTc 
 reto. Determinar o angulo cPq. Ficarei grato se souber a solução, há muito 
 tento e nao consigo resolver.
 Desculpa pela má qualidade da imagem, a fiz no paint, nao tenho muita 
 habilidade.
 Obrigado,
 Adriano Torres
 
 
 =
 Instruções para entrar na lista, sair da lista e usar a lista em
 http://www.mat.puc-rio.br/~nicolau/olimp/obm-l.html
 =
 





This message was sent using IMP, the Internet Messaging Prrogram.
=
Instruções para entrar na lista, sair da lista e usar a lista em
http://www.mat.puc-rio.br/~nicolau/olimp/obm-l.html
=





  _  

Yahoo! doce lar. Faça  
http://us.rd.yahoo.com/mail/br/tagline/homepage_set/*http://br.yahoo.com/homepageset.html
 do Yahoo! sua homepage.


=
Instruções para entrar na lista, sair da lista e usar a lista em
http://www.mat.puc-rio.br/~nicolau/olimp/obm-l.html
=


RE: [obm-l] probleminhas

2006-03-08 Por tôpico João Gilberto Ponciano Pereira
Cheguei em 23...

A lógica que usei é a seguinte Temos que conseguir o menor número das 
unidades. Após isso, basta somar 2 vezes a cota de 2 bombons de 5.

Temos então que achar a combinação de bombons tal que o total seja o mínimo, 
para cada uma das unidades. Temos então:

0 == 0  = 0x5 + 0x7
1 == 21 = 0x5 + 3x7
2 == 12 = 1x5 + 1x7
3 == 33 = 1x5 + 4x7
4 == 14 = 0x5 + 2x7
5 == 05 = 1x5 + 0x7
6 == 26 = 1x5 + 3x7
7 == 07 = 0x5 + 1x7
8 == 28 = 0x5 + 4x7
9 == 19 = 1x5 + 2x7

logo.. como o maior desta lista é o 33, se subtrairmos 10, temos que o maior 
número de bombons que não se pode vender com a combinação de 5 e 7 bombons é 23.


-Original Message-
From: [EMAIL PROTECTED] [mailto:[EMAIL PROTECTED]
Behalf Of Henrique Ren
Sent: Wednesday, March 08, 2006 1:28 PM
To: obm-l@mat.puc-rio.br
Subject: [obm-l] probleminhas


Encontrei esse probleminha e gostaria que alguém me ajudasse a resolvê-lo:

uma doceria venda caixas com 05 e 07 bombons dentro. qual o número máximo de 
bombons que a doceria não consegue vender?
por exemplo: consegue-se vender 17 bombons porém não 11 bombons?

[]s


=
Instruções para entrar na lista, sair da lista e usar a lista em
http://www.mat.puc-rio.br/~nicolau/olimp/obm-l.html
=

=
Instruções para entrar na lista, sair da lista e usar a lista em
http://www.mat.puc-rio.br/~nicolau/olimp/obm-l.html
=


RE: [obm-l] probleminhas

2006-03-08 Por tôpico João Gilberto Ponciano Pereira
4 == 14 = 0x5 + 2x7
 
para 24... 34... 44... basta somar 2n*5 bombons

-Original Message-
From: [EMAIL PROTECTED] [mailto:[EMAIL PROTECTED] Behalf Of Daniel S. Braz
Sent: Wednesday, March 08, 2006 4:01 PM
To: obm-l@mat.puc-rio.br
Subject: Re: [obm-l] probleminhas


24 bombons também não é possível...


Em 08/03/06, João Gilberto Ponciano Pereira  [EMAIL PROTECTED] escreveu: 

Cheguei em 23...

A lógica que usei é a seguinte Temos que conseguir o menor número das 
unidades. Após isso, basta somar 2 vezes a cota de 2 bombons de 5. 

Temos então que achar a combinação de bombons tal que o total seja o mínimo, 
para cada uma das unidades. Temos então:

0 == 0  = 0x5 + 0x7
1 == 21 = 0x5 + 3x7
2 == 12 = 1x5 + 1x7
3 == 33 = 1x5 + 4x7 
4 == 14 = 0x5 + 2x7
5 == 05 = 1x5 + 0x7
6 == 26 = 1x5 + 3x7
7 == 07 = 0x5 + 1x7
8 == 28 = 0x5 + 4x7
9 == 19 = 1x5 + 2x7

logo.. como o maior desta lista é o 33, se subtrairmos 10, temos que o maior 
número de bombons que não se pode vender com a combinação de 5 e 7 bombons é 
23. 


-Original Message-
From: [EMAIL PROTECTED] [mailto: [EMAIL PROTECTED]
Behalf Of Henrique Ren 
Sent: Wednesday, March 08, 2006 1:28 PM
To: obm-l@mat.puc-rio.br
Subject: [obm-l] probleminhas


Encontrei esse probleminha e gostaria que alguém me ajudasse a resolvê-lo: 

uma doceria venda caixas com 05 e 07 bombons dentro. qual o número máximo de
bombons que a doceria não consegue vender?
por exemplo: consegue-se vender 17 bombons porém não 11 bombons?

[]s


=
Instruções para entrar na lista, sair da lista e usar a lista em
http://www.mat.puc-rio.br/~nicolau/olimp/obm-l.html  
http://www.mat.puc-rio.br/~nicolau/olimp/obm-l.html 
=

=
Instruções para entrar na lista, sair da lista e usar a lista em 
http://www.mat.puc-rio.br/~nicolau/olimp/obm-l.html
=




=
Instruções para entrar na lista, sair da lista e usar a lista em
http://www.mat.puc-rio.br/~nicolau/olimp/obm-l.html
=


RE: [obm-l] probleminhas

2006-03-09 Por tôpico João Gilberto Ponciano Pereira
existe uma forma que se eu me recordo eh...no caso de dois numeros X e Y primos 
entre si.. que eh

número maximo = X . Y - ( X + Y )
 
alguem sabe provar isso???
 
Estava pensando nisso... Não consegui provar, mas imagino que seja algo do tipo:
nX mod Y = Rn, com n variando de 1 a Y-1, todos os Rn diferentes entre si, e 
0RnY
 
Suponhamos agora um número M tal que M mod Y = Rm, podemos representá-no na 
forma MX + kY, com M variando de 1 a Y-1 e k 0.
 
Ou seja, supondo M máximo = Y-1, podemos concluir que qualquer número maior ou 
igual a (Y-1)X + 0Y pode ser escrito da forma aX + bY.
 
Entretanto, se N mod Y = Y-1, então N-1 tb poderá ser escrito na forma aX + bY, 
pois neste caso, o coeficiente a será inferior a Y-1 daí chegamos que 
qualquer valor maior que (y-1)x - y pode ser escrito da forma aX + bY, ou seja, 
o maior valor que NÃO pode ser escrito é justamente XY -(X+Y)
 
Será que alguém entendeu essa baboseira toda???
 
 
 
 
 

-Original Message-
From: [EMAIL PROTECTED] [mailto:[EMAIL PROTECTED] Behalf Of Felipe Avelino
Sent: Wednesday, March 08, 2006 4:06 PM
To: obm-l@mat.puc-rio.br
Subject: Re: [obm-l] probleminhas


isso se torna muito cansativo no caso de um numero muito grande...
 
existe uma forma que se eu me recordo eh...no caso de dois numeros X e Y primos 
entre si.. que eh

número maximo = X . Y - ( X + Y )
 
alguem sabe provar isso???
deve envolver teoria combinatoria dos numeros .. não sei ..
 
Em 08/03/06, João Gilberto Ponciano Pereira  [EMAIL PROTECTED] escreveu: 

Cheguei em 23...

A lógica que usei é a seguinte Temos que conseguir o menor número das 
unidades. Após isso, basta somar 2 vezes a cota de 2 bombons de 5. 

Temos então que achar a combinação de bombons tal que o total seja o mínimo, 
para cada uma das unidades. Temos então:

0 == 0  = 0x5 + 0x7
1 == 21 = 0x5 + 3x7
2 == 12 = 1x5 + 1x7
3 == 33 = 1x5 + 4x7 
4 == 14 = 0x5 + 2x7
5 == 05 = 1x5 + 0x7
6 == 26 = 1x5 + 3x7
7 == 07 = 0x5 + 1x7
8 == 28 = 0x5 + 4x7
9 == 19 = 1x5 + 2x7

logo.. como o maior desta lista é o 33, se subtrairmos 10, temos que o maior 
número de bombons que não se pode vender com a combinação de 5 e 7 bombons é 
23. 


-Original Message-
From: [EMAIL PROTECTED] [mailto: [EMAIL PROTECTED]
Behalf Of Henrique Ren 
Sent: Wednesday, March 08, 2006 1:28 PM
To: obm-l@mat.puc-rio.br
Subject: [obm-l] probleminhas


Encontrei esse probleminha e gostaria que alguém me ajudasse a resolvê-lo: 

uma doceria venda caixas com 05 e 07 bombons dentro. qual o número máximo de
bombons que a doceria não consegue vender?
por exemplo: consegue-se vender 17 bombons porém não 11 bombons?

[]s


=
Instruções para entrar na lista, sair da lista e usar a lista em
http://www.mat.puc-rio.br/~nicolau/olimp/obm-l.html  
http://www.mat.puc-rio.br/~nicolau/olimp/obm-l.html 
=

=
Instruções para entrar na lista, sair da lista e usar a lista em 
http://www.mat.puc-rio.br/~nicolau/olimp/obm-l.html
=




=
Instruções para entrar na lista, sair da lista e usar a lista em
http://www.mat.puc-rio.br/~nicolau/olimp/obm-l.html
=


RE: [obm-l] Geometria espacial

2006-03-21 Por tôpico João Gilberto Ponciano Pereira
Estava pensando numa forma mais simples...

Dividir o cubo unitário em 125 cubinhos de lado 1/5

Por casa dos pombos, ao menos um desses cubinhos possui 4 pontos em seu 
interior. E como uma esfera de raio 1/5 contém um cubo de raio 1/5

-Original Message-
From: [EMAIL PROTECTED] [mailto:[EMAIL PROTECTED]
Behalf Of Ronaldo Luiz Alonso
Sent: Tuesday, March 21, 2006 3:22 PM
To: obm-l@mat.puc-rio.br
Subject: Re: [obm-l] Geometria espacial


 Esse problema foi resolvido em uma revista do professor de matemática.
   Vou apenas esboçar como faz ...
   Parece que não mas esse é um problema de química.
Troque cubo unitário por célula unitária e pontos por átomos
Quem não sober o que é cela unitária digite célula unitária no Google.

Eu acredito que a melhor situação seria aquela em que os pontos estão em
em um reticulado (lattice em inglês) uniformemente espaçado.
  Neste caso temos que colocar o maior número de pontos
possíveis dentro deste  reticulado.
   O reticulado então tem que ser um reticulado de Bravais.
 Existem 7 reticulados de Bravais que preenchem o espaço.

http://pt.wikipedia.org/wiki/Rede_de_Bravais

 Para todos esses 7 reticulados, no caso do problema
existem pelo menos 4 pontos dentre os 400 que fazem pate dos vértices
 que estão no interior de uma  esfera de raio 1/5.

Quem não concordar com isso, diga agora ou cale-se para sempre :)

- Original Message - 
From: Dymitri Cardoso Leão [EMAIL PROTECTED]
To: obm-l@mat.puc-rio.br
Sent: Tuesday, March 21, 2006 2:25 PM
Subject: [obm-l] Geometria espacial


* Colocamos 400 pontos, distintos dois a dois, no interior de um cubo
 unitário. Prove que,
 entre os 400 pontos, existem pelo menos 4 que estão no interior de uma
 esfera de raio 1/5.

 Não tenho a menor noçao de como fazer isto, alguém poderia por favor
 resolver?

 _
 Seja um dos primeiros a testar o novo Windows Live Mail Beta. Acesse
 http://www.ideas.live.com/programpage.aspx?versionId=5d21c51a-b161-4314-9b0e-4911fb2b2e6d

 =
 Instruções para entrar na lista, sair da lista e usar a lista em
 http://www.mat.puc-rio.br/~nicolau/olimp/obm-l.html
 =
 

=
Instruções para entrar na lista, sair da lista e usar a lista em
http://www.mat.puc-rio.br/~nicolau/olimp/obm-l.html
=

=
Instruções para entrar na lista, sair da lista e usar a lista em
http://www.mat.puc-rio.br/~nicolau/olimp/obm-l.html
=


RE: [obm-l] Probleminha legal

2006-04-06 Por tôpico João Gilberto Ponciano Pereira
51 rs

-Original Message-
From: [EMAIL PROTECTED] [mailto:[EMAIL PROTECTED] Behalf Of Alexandre Bastos
Sent: Thursday, April 06, 2006 12:11 PM
To: OBM
Subject: [obm-l] Probleminha legal


O emir Abdel Azir ficou famoso por vários motivos. Ele teve mais de 39 filhos, 
incluindo muitos gêmeos. De fato, o historiador Ahmed Aab afirma num dos seus 
escritos que todos os filhos do emir eram gêmeos duplos, exceto 39; todos eram 
gêmeos triplos, exceto 39; todos eram gêmeos quádruplos, exceto 39. O numero de 
filhos do emir é: 



  _  

Abra  
http://us.rd.yahoo.com/mail/br/tagline/mail/*http://br.info.mail.yahoo.com/ 
sua conta no Yahoo! Mail - 1GB de espaço, alertas de e-mail no celular e 
anti-spam realmente eficaz. 


=
Instruções para entrar na lista, sair da lista e usar a lista em
http://www.mat.puc-rio.br/~nicolau/olimp/obm-l.html
=


[obm-l] RE: [obm-l] Questão de concurso

2006-04-10 Por tôpico João Gilberto Ponciano Pereira
Dica... Veja que não se fala de quíntuplos, sextuplos, etc. 
 
Chamando:
2X o número de filhos gêmeos duplos;
3Y o número de filhos gêmeos triplos;
4Z o número de filhos gêmeos quádruplos;
R o resto (quítuplos, sextuplos, etc...)
 
 
Então.. da primeira frase, temos:
3Y + 4Z + R = 39
da segunda:
2X + 4Z + R = 39
e da última:
2X + 3Y + R = 39
 
Daí, basta achar a solução onde X, Y, Z e R sejam inteiros não negativos...
 
 

-Original Message-
From: [EMAIL PROTECTED] [mailto:[EMAIL PROTECTED] Behalf Of Alexandre Bastos
Sent: Monday, April 10, 2006 10:48 AM
To: OBM
Subject: [obm-l] Questão de concurso


Bom dia, amigos. Alguém pode me ajudar com esse probleminha?
 
O emir Abdel Azir ficou famoso por vários motivos. Ele teve mais de 39 filhos, 
incluindo muitos gêmeos. De fato, o historiador Ahmed Aab afirma num dos seus 
escritos que todos os filhos do emir eram gêmeos duplos, exceto 39; todos eram 
gêmeos triplos, exceto 39; todos eram gêmeos quádruplos, exceto 39. O numero de 
filhos do emir é:



  _  

Abra  
http://us.rd.yahoo.com/mail/br/tagline/mail/*http://br.info.mail.yahoo.com/ 
sua conta no Yahoo! Mail - 1GB de espaço, alertas de e-mail no celular e 
anti-spam realmente eficaz. 


=
Instruções para entrar na lista, sair da lista e usar a lista em
http://www.mat.puc-rio.br/~nicolau/olimp/obm-l.html
=